You are on page 1of 80

2 Mechanics

2.1 Motion

Answers to 2.1 Motion (Skills worksheet)

1) Time: 16.8 s; Velocity: 53.1 ms-1

2) Speed: 324 m

3) Distance: 200 m; Average speed: 9.4 ms-1

4) The truck arrives 0.104 hours earlier than the car.

5) Time: 40 s

6) They collide 3.3 m from the starting position of the first ball.

7) Time: 5.0 s

8)
a) Acceleration: 2 ms-2
b) Displacement: 160 m
c) Average velocity: 11.4 ms-1

9) Times: 0.18 s and 1.1 s

10) Time: 3.8 s

© Oxford University Press 2015


1
www.oxfordsecondary.co.uk/acknowledgements
1
2 Mechanics

2.2 Forces

Answers to 2.2 Forces (Skills worksheet)

1) Force: 14 N

2) Force: 45 N

3) Mass: 9.7 kg

4) Coefficient of dynamic friction: 0.206

5) Yes, it will slide

6) Acceleration: 0.11ms-2 [S24oW]

© Oxford University Press 2015


1
www.oxfordsecondary.co.uk/acknowledgements
2
2 Mechanics

2.3 Work, energy and power



Answers to 2.3 Energy transformations (Skills worksheet)

1)
a) Gravitational potential energy: 353 J
b) Gravitational potential energy: 235 J
c) Kinetic energy: 118 J
d) Gravitational potential energy: 177 J
e) Kinetic energy: 177 J
f) Gravitational potential energy: 0 J
g) Kinetic energy: 353 J

2) The total is always the same.

3) Speed: 0.86 ms-1

4) Work: 460 J

5) Height: 0.90 m

6) Speed: 15 ms-1

© Oxford University Press 2015


1
www.oxfordsecondary.co.uk/acknowledgements
3
2 Mechanics

2.3 Work, energy and power



Answers to 2.3 Work, power and efficiency (Skills worksheet)

1) Work: 2200 J; Power: 180 W

2) Work: 24 J; Velocity: 2.0 ms-1

3) Output power: 3200 W

4) Efficiency: 85%

© Oxford University Press 2015


1
www.oxfordsecondary.co.uk/acknowledgements
4
2 Mechanics

2.4 Momentum

Answers to 2.4 Momentum and impulse (Skills worksheet)

1) Impulse: 71 000 Ns

2) Velocity: 0.4 ms-1

3) Velocity: 3.7 ms-1

4) Mass: 23 kg; Velocity: 1.8 ms-1

5) Velocity: 0.37 ms-1 in the opposite direction that he threw the rocks.

© Oxford University Press 2015


1
www.oxfordsecondary.co.uk/acknowledgements
5
Constant acceleration
1. A body travelling at 5ms-1 accelerates at 20ms-2 for 100m. What will its final velocity be?

2. A body travelling at 2ms-1 accelerates to 8ms-1 in 2s. Find its displacement.

3. A body is thrown downwards with a velocity of l0ms-1. Assuming the acceleration due to gravity is
10ms-2 calculate its displacement after 2s?

4. A boy standing on the edge of a cliff throws a stone upwards with a velocity of 5ms-1. If the cliff is
50m high how much time does it take for the stone to hit the bottom of the cliff?

6
Distance, Displacement and Speed

1. A person walks up a hill as shown in the diagram.

a) How many meters does the person walk?

2. A person swims across a river as shown in the diagram.

a) How many seconds does the swimmer take to cross the


river?

b) How far downstream will the swimmer be swept in the time


taken to cross the river?

c) What is the final displacement of the swimmer?

3. The distance from Flekke to Dale is 6km on the line shown on


the map. The direction is 20° East of North.

a) How far to the North of Flekke is Dale?

© Chris Hamper, InThinking


www.physics-inthinking.co.uk 1
7
Energy and Power
1. A 2kg ball rolls down the slope shown. Calculate the loss of PE.

2. A 2kg ball is pushed onto a spring of spring constant 100N/cm as shown. (It’s a very strong
finger by the way)
(a) Calculate elastic PE stored in the spring

(b) Calculate the maximum height reached by the ball

3. A machine is used to lift a 50kg mass 4m vertically in 10s.


(a) Calculate the increase in PE

(b) Calculate the power of the machine.

Formulae
KE = ½mv2
PE=mgh
EPE=½kx2
P=E/t

© Chris Hamper, InThinking


www.physics-inthinking.co.uk 1

8
Forces
1. A Helium balloon is held stationary with a string as shown. Draw and name all the forces
acting on the balloon.

2. A box rests on a slope as shown in the diagram. Draw and label the
forces acting on the box.

3. Two perpendicular ropes are use to pull a load along the ground as shown in the diagram.
Calculate the resultant force.

4. A Force acts at a 30° angle to the horizontal as shown.


Calculate the Horizontal component of the force.

9
Graphs of Motion
1. A car travelling at 10m/s accelerates at a constant rate for 20s until its velocity is 40m/s.

v (m/s)

50
40
30
20
10

t (s)
10s 20s

a. Using the axis above draw v-t graph for this motion.
b. Use the graph to calculate the distance traveled

c. Use the graph to calculate the acceleration.

2. The motion of a car travelling a long a straight road can be split into 3 stages:
(i) 30s of constant acceleration from rest.
(ii) 60s of constant velocity
(iii) 10s of braking to stop
Draw displacement – time, velocity – time and acceleration – time graphs for this motion

10
Newton’s 1st
1. State Newton’s 1st law.

2. A freefall parachutist falls with constant velocity.


(a) Label the Forces acting on the parachutist in the diagram.

(b) What can you deduce about the size of these forces?

3. A plastic football is held underneath the water in a swimming pool and released. The mass of
the ball is 500g and the upthrust it experiences is 40N.
(a) Draw the forces on the ball in the diagram.
(b) What is the resultant force?

(c) What can you deduce about the motion of the ball?

4. Calculate the momentum of a 1200kg car travelling at 20ms-1.

Formulae
P=mv

© Chris Hamper, InThinking


www.physics-inthinking.co.uk 1

11
Newton’s 2nd
1. State Newton’s 2nd law of motion.

2. A man is standing in a lift that is accelerating upwards at a rate of


2ms-2.
(a) Draw the forces acting on the man

(b) If the mass of the man is 60kg calculate the Normal Force acting
on his feet.

3. Forces act on the 50kg mass as shown


(a) Calculate the acceleration of the mass.

(b) If the original momentum is zero calculate the momentum of the box after 2s.

Formulae
F=ma

© Chris Hamper, InThinking


www.physics-inthinking.co.uk 1

12
Newton’s 3rd
1. State Newton’s 3rd law of motion.

2. When a car accelerates the friction of the tyres push the ground backwards. Use Newton’s
3rd law to deduce what happens to the car.

3. Two balls collide as shown in the diagram. Use the principle of conservation of momentum
to calculate the velocity of the big ball.

4. Calculate the impulse of the big ball in the previous question.

Formulae
p=mv
Impulse = ∆mv

© Chris Hamper, InThinking


www.physics-inthinking.co.uk 1

13
Projectiles
1. A ball is projected as shown in the diagram.
Formulae
1
! = #$ + ($ )
2

* ) = #) + 2(!
*−#
(=
$

(# + *)
!= $
2
(a) Draw a line showing the possible path of the ball.
(b) Calculate the vertical component of the balls velocity.

(c) Using your answer to (b) calculate the time of flight.

(d) Calculate the horizontal velocity of the ball.

(e) Using your answer to (c) and (d) calculate the balls range.

2. The ball above is projected with the same angle and velocity from a 10m high sea cliff.
Calculate the time taken for the ball to hit the sea.

© Chris Hamper, InThinking


www.physics-inthinking.co.uk 1

14
15
E N D - O F -TO P I C Q U E S T I O N S

Solutions for Topic 2 – Mechanics


1. a) P V

A
b) horizontal speed = 15 × cos 45 = 10.6 m s–1
vertical speed = 15 × sin 45 = 10.6 m s–1 upwards
v2 = u2 + 2as; v2 = 10.62 + 2 × 9.8 × 25 = 112 + 490 = 602
v = ± 24.5 m s–1 (positive value is correct one to use)
___________
so speed is √10.62 + 24.52
= 27 m s–1
2. a) (i) h = _ v2 = 3.2 m
2g
u
(ii) t = _
g = 0.80 s
b) time to go from top of cliff to the sea = 3.0 – 1.6 = 1.4 s
s = 8.0 × 1.4 + 5.0 × (1.4)2 = 21 m;
3. travels vertically 1.25 m in 0.5 s;
g=_ 2s
t2
to give g = 10 (±1) m s–2
4. a) (i) Zero
(ii)

(normal) reaction (normal) reaction

force downwards on pedals


weight/gravity force/mg

(iii) The drag force is equal to the forward force; the net force is zero and therefore the
acceleration is zero.
resistive force _
b) (i) acceleration = __ mass = 40 = 0.57 m s–2
70
(ii) v2 = u2 + 2as; 0 = 64 – (2 × 0.57 × s); s = 56 m
(iii) air resistance or bearing friction or effectiveness of brakes depends on speed; air resistance
reduced as speed drops, estimate will be too low, stopping distance will be further
5. The net force on the car is 0.3 × 1000 = 300 N. There is an additional drag force of 500 N.
T = 300 + 500 = 800 N.
6. T1 sin 60 = T2 sin 30
T1 cos 60 + T2 cos 30 = 3800
T1 = 1900 N; T2 = 3300 N

16
© Oxford University Press 2014: this may be reproduced for class use solely for the purchaser’s institute 1

839213_Solutions_Ch02.indd 1 12/17/14 4:09 PM


E N D - O F -TO P I C Q U E S T I O N S

7. a) power is 0.66 kW (read off from graph)


b) power = frictional force × speed
660 = 330 N
force = _
2
8. a) use the area under the graph as this is v × t
b) (i)
drag force
ball at t = 2.0s

weight

Earth’s surface

(ii) 25

20
v/m s-1

15

10

0
0 2 4 6 8 10
t/s
25 – 6
the acceleration of the ball is equal to the gradient of the graph gradient = _
4.8 – 0
= 4.0 m s–2
(iii) The net force on the ball is 2 N, the weight is 4.9 N, so the difference between these is the
magnitude of the drag force = 2.9 N.
(iv) At 5.0 s the gradient is smaller and therefore the acceleration is less than at 2.0 s. The
weight is constant and therefore the drag force is greater.
c) gain in kinetic energy = _1 × 0.5 × 252 = 156 J
2
loss in gravitational potential energy = 0.5×9.8×190 = 931 J
change (loss) in energy = 931 – 156 = 775 J
9. a) (i) normal reaction/N

driving force/
thrust/OWTTE

friction/drag/
air resistance weight/gravity force/W
(do not accept gravity)

(ii) zero
output power 70
b) input power = __ = _
efficiency 0.35
= 200 kW
c) height gained in 1 s = 6.2 sin(6) = 0.648 (m)
rate of change of PE = 8.5 × 103 × 9.81 × 0.648
= 5.4 × 104 W
p
d) F = _
v =
_ ()
1.6 × 104
6.2
= 2.6 kN

17
© Oxford University Press 2014: this may be reproduced for class use solely for the purchaser’s institute 2

839213_Solutions_Ch02.indd 2 12/17/14 4:09 PM


E N D - O F -TO P I C Q U E S T I O N S

10. a) (i) momentum before = 800 × 5 = 4 000 N s


momentum after = 2 000v
conservation of momentum gives v = 2.0 m s–1
(ii) KE before = 400 × 25 = 10 000 J KE after = 1 000 × 4 = 4 000 J
loss in KE = 6 000 J;
b) transformed / changed into heat (internal energy) and sound
11. a) momentum of object = 2 × 103 × 6.0
momentum after collision = 2.4 × 103 × v
use conservation of momentum, 2 × 103 × 6.0 = 2.4 × 103 × v
v = 5.0 m s–1
b) KE of object and bar + change in PE = 0.5 × 2.4 × 103 × 25 + 2.4 × 103 × 10 × 0.75
use ∆E = Fd, 4.8 × 104 = F × 0.75
F = 64 kN
12. a) time = ___ 81 = 4.8 × 107 s
2.2 × 10–25 × 77 × 1018
b) rate of change of momentum of the xenon atoms
= 77 × 1018 × 2.2 × 10–25 × 3.0 × 104
= 0.51 N
= mass × acceleration
where mass = (540 + 81) kg
acceleration of spaceship = _0.51
621
= 8.2 × 10–4 m s–2
c) a = _F
m
since m is decreasing with time, then a will be increasing with time
d) change in speed = area under graph
= (8.0 × 4.8) × 102 + _1 (4.8 × 1.4) × 102
2
final speed = (8.0 × 4.8) × 102 + _1 (4.8 × 1.4) × 102 + 1.2 × 103 5.4 × 103 m s–1
2
(350 × 2.62)
__
13. a) centripetal force = = 410 N
5.8
tension = 410 + (350 × 9.8) = 3800 N
b) idea of use of area under graph
distance = _1 × 0.15 × 2.6
2
= 0.195 m
c) idea of momentum as mv
total change (= 2.6 × 350) = 910 N s

18
© Oxford University Press 2014: this may be reproduced for class use solely for the purchaser’s institute 3

839213_Solutions_Ch02.indd 3 12/17/14 4:09 PM


IB Physics
Assessment paper answers: 2 Mechanics

1. A (1)
2. A (1)
3. D (1)
4. C (1)
5. B (1)
6. D (1)
7. D (1)
8. C (1)
9. A (1)
10. B (1)
Total: 10 marks

1. a)

Mark both together.


VH: horizontal arrows equal in length;
VV: two vertical arrows, the one at 1.0 m noticeably longer than the one at
0.5 m; (2)
If arrows correct, but wrong points award (1) only.
b) curve that goes through all data points;
stops at y = 1.8 m as this is the height of the wall;
from graph d = 1.5 (±0.1) m; (3)

© Oxford University Press 2015


www.oxfordsecondary.co.uk/acknowledgements 1
19
IB Physics
Assessment paper answers: 2 Mechanics

c) travels vertically 1.8 m in 0.6 s /1.25 m in 0.5 s;


g = (2s / t2);
to give g = 10 (± 1) m s-2; (3)
Award (2) marks maximum for any time shorter than 0.5 s.
Total: 8 marks

2. a) a straight line; through the origin; (2)


b) any straight line; that fits within all the error bars; (2)

c) (i) a systematic error is when every data point deviates from the
“correct” value;
by the same fixed amount as seen by intercept on graph; (2)
Accept answers that explain by giving an example of a possible systematic
error (e.g. friction).
(ii) 0.3 N; (1)
Accept 0.25 N to 0.35 N; watch for use of wrong axis.
Total: 7 marks

3. a) (i) attempt using principle of moments (even if in error);


F x 1.2 = 600 x 0.4;
therefore F = 200 N; (3)
(ii) resultant force = zero, therefore reaction = 600 – 200 = 400 N;
up; (2)

© Oxford University Press 2015


www.oxfordsecondary.co.uk/acknowledgements 2
20
IB Physics
Assessment paper answers: 2 Mechanics

b) (i) correct use of Fhorizontal = 260 cos40 or 260 sin50;


to give Fhorizontal = 1992 N 200 N; (2)
(ii) realization that resultant force is zero (constant velocity);
so friction = Fhorizontal 200 N; (2)
Total: 9 marks

4. a) mass x velocity; (1)


b) (i) momentum before = 800 x 5 = 4000 N s;
momentum after = 2000ν;
conservation of momentum gives ν = 2.0 m s-1; (3)
(ii) KE before = 400 x 25 = 10 000 J
KE after = 1000 x 4 = 4000 J;
loss in KE = 6000 J; (2)
Total: 6 marks

© Oxford University Press 2015


www.oxfordsecondary.co.uk/acknowledgements 3
21
Answers to exam-style questions
Topic 2
Where appropriate, 1 ✓ = 1 mark
1 D
2 C
3 C
4 D
5 A
6 D
7 D
8 A
9 C
10 A
11 a i The equation applies to straight line motion with acceleration g. Neither condition is satisfied here. ✓
ii This equation is the result of energy conservation so it does apply since there are no frictional forces present. ✓
v2 4.8 2
b From v = 2 gh we find h = = = 1.174 ≈ 1.2 m. ✓
2 g 2 × 9.81
1 1
c i The kinetic energy at B is E = mv 2 = × 25 × 4.8 2 = 28.8 J. ✓
2 2
The frictional force is f = µ K N = µ Kmg = 0.45 × 25 × 9.81 = 110.36 N and so the work done by this
force is the change in the kinetic energy of the block, and so 110.36 × d = 28.8 ⇒ d = 0.261 ≈ 0.26 m . ✓
f 110.36
ii The deceleration is = = 4.41 m s −2, ✓
µ 25
and so 0 = 4.8 − 4.41 × t giving 1.1 s for the time. ✓

d The speed at B is independent of the mass. ✓


1 2 1 2 v2
fd = mv ⇒ µ Kmgd = mv ⇒ d = ,✓
2 2 2µ K
and so the distance is also independent of the mass. ✓
12 a i v x = v cos θ = 22 × cos 35° = 18.0 ≈ 18 m s −1 ✓
v y = v sin θ = 22 × sin 35° = 12.6 ≈ 13 m s −1 ✓
ii Graph as shown. ✓
vx / m s–1 20

15

10

0
0 1 2 3 4
t/s

PHYSICS FOR THE IB DIPLOMA © CAMBRIDGE UNIVERSITY PRESS 2015 ANSWERS TO EXAM-STYLE QUESTIONS – TOPIC 2 1
22
Graph as shown. ✓
vy / m s–1 20

10

0 t/s
1 2 3 4

–10

–20

–30
b i At maximum height: v y2 = 0 = uy2 − 2 gy . ✓
uy2
y= ✓
2g
12.6 2
and so y = = 8.1 m ✓
2 × 9.8
OR
v y = 0 = v sin θ − gt 12.6 − 9.8t = 0 ✓
so t = 1.29 s ✓
1
Hence y = 12.6 × 1.29 − × 9.8 × 1.29 2 = 8.1 m ✓
2
ii The force is the weight, i.e. F = 0.20 × 9.8 = 1.96 ≈ 2.0 N. ✓
1 2 1
c i mu + mgh = mv 2 hence v = u 2 + 2 gh ✓
2 2
v = u 2 + 2 gh = 22 2 + 2 × 9.8 × 32 = 33.3 ≈ 32 m s −1 ✓

ii v 2 = v x2 + v y2 ⇒ v y = − v x2 − v x2 = − 33.32 − 18.0 2 = −28.0 m s −1 ✓


Now v y = uy sin θ − gt so −28.0 = 12.6 − 9.8 × t hence t = 4.1 s ✓
d i Smaller height. ✓
Smaller range. ✓
Steeper impact angle. ✓
y

ii The angle is steeper because the horizontal velocity component tends to become zero. ✓
Whereas the vertical tends to attain terminal speed and so a constant value. ✓
13 a i In 1 second the mass of air that will move down is ρ(π R 2v ). ✓
The change of its momentum in this second is ρ(π R 2v )v = ρπ R 2v 2. ✓
∆p
And from F = this is the force. ✓
∆t

2 ANSWERS TO EXAM-STYLE QUESTIONS – TOPIC 2 PHYSICS FOR THE IB DIPLOMA © CAMBRIDGE UNIVERSITY PRESS 2015
23
ii ρπ R 2v 2 = mg ✓
mg 0.30 × 9.8
And so v = 2 = = 3.53 ≈ 3.5 m s −1. ✓
ρπ R 1.2 × π × 0.252
b The power is P = Fv where F = ρπ R 2v 2 is the force pushing down on the air and so P = ρπ R 2v 2. ✓
So P = 1.2 × π × 0.252 × 3.532 = 2.936 ≈ 3.0 W ✓
c i From F = ρπ R 2v 2 the force is now 4 times as large, i.e. 4mg and so the net force on the helicopter is 3mg. ✓
1 2s
And so the acceleration is constant at 3g. Hence s = × 3 g × t 2 ⇒ t = ≈ 0.90 s. ✓
2 3g
2s
ii v = 3 gt = ✓
3g
v ≈ 26 m s −1 ✓
iii The work done by the rotor is W = Fd = 4mgd = 4 × 0.30 × 9.8 × 12 = 141 J. ✓
14 a i The area is the impulse i.e. 2.0×103 N s. ✓
ii The average force is found from F ∆ t = 2.0 × 10 3 N s. ✓
2.0 × 10 3
And so F = = 1.0 × 104 N. ✓
0.20
1.0 × 104
Hence the average acceleration is a = = 1.25 × 10 3 m s −2. ✓
8.0
iii The final speed is v = at = 1.25 × 10 3 × 0.20 = 250 m s −1. ✓
And so the average speed is 125 m s −1. ✓
1 2 1 3 2
iv s = at = × 1.25 × 10 × 0.20 ✓
2 2
s = 25 m ✓
b i The final speed is v = at = 1.25 × 10 3 × 0.20 , ✓
v = 250 m s −1 . ✓
1 1
ii The kinetic energy is E K = mv 2 = × 8.0 × 250 2 ✓
2 2
E K = 2.5 × 105 J ✓
E K 2.5 × 105
iii P = = ✓
t 0.20
P = 1.25 × 106 W ✓

15 a i It is zero (because the velocity is constant). ✓


ii F − mg sin θ − f = 0 ✓
F = mg sin θ + f = 1.4 × 104 × sin 5.0° + 600 ✓
F = 1820 N ✓
b The power used by the engine in pushing the car is P = Fv = 1820 × 6.2 = 1.13 × 104 W, ✓
P = 11.3 kW. ✓
11.3
The efficiency is then e = = 0.75 ✓
15

PHYSICS FOR THE IB DIPLOMA © CAMBRIDGE UNIVERSITY PRESS 2015 ANSWERS TO EXAM-STYLE QUESTIONS – TOPIC 2 3
24
c i Initial speed zero. ✓
Terminal speed. ✓
v

time
ii Initial acceleration not zero. ✓
And approaching zero. ✓
a

time
16 a i The change in momentum is ∆ p = 0.090 × (90 − 130), ✓
∆ p = −3.6 N s . ✓
ii This is also the negative change in the momentum of the block and so 1.20v = 3.6 N s
−1
giving v = 3.0 m s . ✓
1 1
iii The initial kinetic energy is E = mv 2 = × 0.090 × 130 2 = 422.5 J. ✓
2 2
1 1
The final kinetic energy is E = × 0.090 × 90 2 + × 1.20 × 3.0 2 = 369.9 J. The change is then
2 2
∆E = 369.9 − 422.5 = −52.6 ≈ −53 J. ✓
1
b We have conservation of energy and so × m × 3.0 2 = m × 9.8 × h and so h = 0.459 m. ✓
2
But h = L − L cos θ and so 0.459 = 0.80 × (1 − cos θ ) ✓
giving cos θ = 0.426 and so θ = 64.77° ≈ 65° ✓
c i It is not because there is a net force on it. ✓

mg

v2
ii From the diagram, T − mg cos θ = m .✓
L
But v = 0 and so T = mg cos θ = 1.20 × 9.8 × cos 64.77° = 5.0 N . ✓
T = 5.0 N. ✓

4 ANSWERS TO EXAM-STYLE QUESTIONS – TOPIC 2 PHYSICS FOR THE IB DIPLOMA © CAMBRIDGE UNIVERSITY PRESS 2015
25
1 2 1 2 1 3 1 4 1 2 2 1 1

1 2 1 2
d3 2
g 3t1 9 d1 , d 4 2
g 4t1 16d1 , etc.

17. The elevator moving from the second floor to the fifth floor is NOT an example of constant
acceleration. The elevator accelerates upward each time it starts to move, and it accelerates
downward each time it stops.

Topic 2.1a Kinematics Problems


Ignoring air resistance, a rock falling from a cliff would have a constant acceleration. (If air
resistance is included, then the acceleration will be decreasing as the rock falls.) A dish resting on a
table has an acceleration of 0, so the acceleration is constant.

Conceptual Questions
18. As an object rises WITH air resistance, the acceleration is larger in magnitude than g, because both
gravity and air resistance will be causing a downward acceleration. As the object FALLS with th air
Giancoli Physics: Principles with Applications, 6 Edition
(These questions are not in an IB style but instead designed to check your understanding of the concept of this topic. You should
resistance, the acceleration will be smaller in magnitude than g, because gravity and resistance will
try your best to appropriately communicate your answer using prose)
be opposing each other. Because of the smaller acceleration being applied over the same distance,
7. the
If anreturn
objectspeed will be to
is traveling slower than but
the north the slowing
launch speed. down, it has a northward velocity and a southward
1. Can an object have zero velocity and nonzero acceleration at the same time? Give examples.
acceleration.
19. If an object is at the instant of reversing direction (like an object thrown upward, at the top of its
th
Giancoli
8. path), it instantaneously
The velocity of an objecthas canabezero velocity
negative whenand its
a non-zero Physics:
acceleration isPrinciples
acceleration
positive. with
at theIf we Applications,
same time.theA6positive
define Edition
person
at the exact bottom of a “bungee” cord plunge also has an instantaneous
direction to be to the right, then an object traveling to the left that is having a reduction in speed will velocity of zero but a non-
zero
have (upward)
a negativeacceleration
velocity with at the same time.
a positive acceleration.
7. If an object is traveling to the north but slowing down, it has a northward velocity and a southward
20. An If again
acceleration.
object wemoving
define with
the positive
a constantdirection
velocity to behastoa the right, then
non-zero an object
velocity and a traveling to the right
zero acceleration that is
at the
2. Can havingthe avelocity
reduction of inan object
speed will be negative
have a positive when its
velocity acceleration
and a negative
same time. So a car driving at constant speed on a straight, level roadway would meet this condition.
is positive?
acceleration. What about vice-versa?
8. The velocity of an object can be negative when its acceleration is positive. If we define the positive
9.
21. TheIf north
direction
object istodefined
starts as the
be to with
the right,positive
then an
a constant direction,
object in
velocity then
traveling an to
object
the positive traveling
that isAtto
thedirection.
left the south
having
about ands, increasing
at =reduction
17 whenin speed inwill
the object
speed
have ahas both
negative a negative
velocity velocity
with a and
positive a negative
acceleration. acceleration. Or,
is at the 5 meter position, it begins to gain speed – it has a positive acceleration. At about t = 27 s,if up is defined as the positive
direction,
when thewe then anisobject
object at about falling duemto
the direction
12 gravity ithas
position, both to a negative velocity and a negative
If again
acceleration. define the positive to be tobegins
the right, slow
then down
an object – it traveling
has a negative
to theacceleration.
right that is
The
having a reduction in speed will have a positive velocity and a negative acceleration. the origin of
object instantaneously stops at about t = 37 s, reaching its maximum distance from
20 m. The object then reverses direction, gaining speed while moving backwards. At about t = 47 s,
10. when
If the two cars emerge
theisobject is as
again sideat by side,
about thethen
12 mtheposition,
one moving fasterstarts
the object is passing
to the
slowthedown,
other and
one.appears
Thus car to A
9. is If north
passing defined
car the positive direction, then an object traveling to south and increasing in
3. Can stop athas
an tobject50 s,B.
= both a10
be
With
m from the acceleration
increasing the in
starting data given for the problem, the ensuing motion would be that
andapoint.
speed as itsbut
acceleration decreases? If defined
so, up give
speed
car A would pull negative
away from velocity
car B for atime,
negative acceleration.
eventually car Or, if up iscatch
B would toasan
the
and example.
positive
pass If not,
car A.
explain.
direction, then an object falling due to gravity has both a negative velocity and a negative
22. Initially,
acceleration. the object moves in the positive direction with a constant acceleration, until about t = 45 s,
11. when
Assume thatanorth
it has velocityis theof positive
about 37direction.
m/s in theIfpositivea car isdirection.
moving south The and gaining speed
acceleration at an
then decreases,
increasing rate, then the acceleration will be getting larger in magnitude. However, since the speed
10. reaching
If the twoancars
acceleration
instantaneous
is emerge side
directed
acceleration
by side, then
southwards, the
ofthe0 atoneabout
acceleration movingt = 50faster
is
s, when
negative, isandthe object
passing
is
has itsone.
the other
getting more
maximum
Thus car
negative. A
That
of
is about 38car
passing m/s.
B. The
With object
the then beginsdata
acceleration to slow
given down,
for thebutproblem,
continues the toensuing
move inmotion the positive
would be that
is a decreasing acceleration as the speed increases.
car A would pull away from car B for a time, but eventually car B would catch up to and pass car A.
© 2005 Pearson Education, Inc., Upper Saddle River, NJ. All rights reserved. This material is protected under all copyright laws as they
Another
currently exist. Noexample would
portion of this materialbemay
an be
object falling
reproduced, in anyWITH
form orair resistance.
by any Aspermission
means, without the object falls,from
in writing it gains
the speed,
11. the
publisher. air resistance
Assume that northincreases.
is the positiveAs the air resistance
direction. increases,
If a car is moving thesouth
acceleration
and gaining of thespeed
fallingat object
an
decreases,
increasing and rate,itthen
gains thespeed less quickly
acceleration will be 14 larger
thegetting
longer it falls.in magnitude. However, since the
acceleration is directed southwards, the acceleration is negative, and is getting more negative. That
12. Assuming
is a decreasing that the catcher catches
acceleration the ball
as the speed at the same height at which it left the bat, then the ball will
increases.
be traveling with a speed of 120 km/h when caught. This is proven in problem 41.
Another example would be an object falling WITH air resistance. As the object falls, it gains speed,
13. As the aairfreely
resistance
fallingincreases.
object speeds As theup,airitsresistance
acceleration increases, the acceleration
due to gravity stays the same.of the falling object is
If air resistance
decreases, and
considered, thenit the
gains speed less of
acceleration quickly the longer
the object is dueittofalls.
both gravity and air resistance. The total
acceleration gets smaller as the object speeds up, until the object reaches a terminal velocity, at
12. which
Assuming
timethat the catcher
its total catches
acceleration is the ballThereafter
zero. at the same
its height at whichconstant.
speed remains it left the bat, then the ball will
be traveling with a speed of 120 km/h when caught. This is proven in problem 41.
14. To estimate the height, throw the ball upward and time the flight from throwing to catching. Then,
13. As a freely falling object speeds up, its acceleration due to gravity stays the same. If air resistance is
ignoring air resistance, the time of rising would be half of the time of flight. With that "half" time,
considered, then the acceleration of the object is due to both gravity and air resistance. The total
assuming that the origin is at the top of the path and that downward is positive, knowing that the ball
acceleration gets smaller as the object speeds up, until the object reaches1 a terminal velocity, at
started
which timefromits thetotal
top acceleration
of the path with a speed
is zero. of 0, useitsthe
Thereafter equation
speed remains gt 2 with that time and the
y constant.
2

acceleration due to gravity to find the distance that the ball fell. With the same "half" time, we know
14. that at the top
To estimate theofheight,
the path, the speed
throw the ballis upward
0. Taking andthe upward
time direction
the flight from as positive,
throwing to use the equation
catching. Then,
ignoring
v v0 at air resistance,
0 vthe 0
time
gt of rising
v0 would be half
gt to find of the time
the throwing of flight. With that "half" time,
speed.
assuming that the origin is at the top of the path and that downward is positive, knowing that the ball
started
15. The from the
average speedtopisofNOT
the path with aSince
80 km/h. speedtheof two
0, use the equation
distances traveled 1
y were
2
gt 2the
with thatthe
same, time and of
times the
travel were unequal. The time to travel from A to B at 70 km/h is longer than
acceleration due to gravity to find the distance that the ball fell. With the same "half" time, we knowthe time to travel from
B to C at 90 km/h. Thus we cannot simply average the speed numbers. To find
that at the top of the path, the speed is 0. Taking the upward direction as positive, use the equation the average speed,
we
v need
v0 at to calculate 0 (total
v0 distance)
gt v/ 0(total
gt time).
to findWetheassume
throwing thespeed.
distance from A to B and the
distance from B to C are both d km. The time to travel a distance d with a speed v is t = d / v.
15. The average speed is NOT 80 km/h. Since the two 26distances traveled were the same, the times of
travel were unequal. The time to travel from A to B at 70 km/h is longer than the time to travel from
Graph of the velocity Graph of the acceleration
Giancoli Physics: Principles with Applications, 6th Edition
6
30
5

20Topic 2.1b Kinematics Problems


25
4

a (m/s 2)
Graph of the velocity Graph of the acceleration
v (m/s)

15 3
6
30
10 2
5
25
5 1
Calculation-based Questions 4

a (m/s 2)
20
0 0
v (m/s)

15 0 1 2 3 4 5 6 3 0 1 2 3 4 5 6
1. A car
accelerates
10 from
t (s)13m/s
to 25m/s in 6.0s. What was its acceleration?
t (s) How far did it travel in
2

this5time? Assume constant acceleration. 1


0
0 1 2 3 4 5 6
0
[2 marks]
0 1 2 3 4 5 6
v v0 25 m s 13 m s 2
21. By2definition, the
Chapter acceleration
t (s) is a 2.0 .
m stKinematics
(s)
t 6.0 Describing
s Motion: in One Dimension
The distance of travel can be found from Eq. 2-11b.
2
33. Choose x downward
x0 v0 t to 1
at 2 13 m s 6.0
2 be the positive direction,
1
v sv0 2 25 2.0mm
and
2
s s 13
take 6.0s0s at the114
y 0m topm of the cliff. The initial
2
21. By definition, the acceleration is a 2.0 m s .
velocity is v0 0 , and the acceleration is t a 9.80 m 6.0s s2 . The displacement is found from equation
2. A(2-11b),
22. cardistance
The
The slows
withdown
acceleration of from
can
travel
x replaced
be
can 23m/s
found
bybe to rest
from
y. found from in a distance
Eq. (2-11c).
Eq. 2-11b. of 85m. What was the acceleration, assumed
constant? 2 21 2
1 2
v v 2 2.00m 2s23 2
m6.0s s 22 114 m 2
yvx2 xy0v002 vv020tta 2x12 at
atx2 0 13 m y as 0 6.0
0 s 12 09.80 m s 3.25 s 3.1my s 51.8
. m
2 x x0 2 85 m [1 mark]
22. The acceleration can be found from Eq. (2-11c).
34. v0 0 , the final velocity is
23. Choose
Assumedownwardthat the plane to bestarts
the positive
from rest. direction.
The runway The initial
distance velocity
is2 found is by solving Eq. 2-11c for
2 2 v 2 v02 0 23 m s
x x0v. v0 2 a 1m x xs 0 a 3.1m s 2 .2
v 85 km h 23.61m2 sx, and 2 85 m is a 9.80 m s . The time can be
x0 the acceleration
3.6 km h v 2
v 2
33 m s
2
0
2 2
found vby solving
v0 2 aEq. x 2-11a
x0 for thex time. x0 0
2
1.8 10 2 m
23. Assume that the plane starts from rest. The runway 2a distance
2 3.0 m iss found by solving Eq. 2-11c for
3. Estimate
x xv0 . vhow atlong it ttook King
v v0 Kong 23.61m to falls straight
0
2.4 down
s from the top of the Empire State Building
24. (380m high) starts
and his velocity a just before
9.80 m2he s touched the ground. Ignore air resistance.
0 2
The sprinter from rest. The average vaccelerationv02 33ismfound
s
2
from
0 Eq. 2-11c.
v 2 2
v0 2 a x x0 x x0 2 2 1.8 10 2 m [2 marks]
2 2 v v022 a 11.5 2 m
3.0 sm s 20 2 2
35. Choose v downward
v0 2 a to x be a direction, and take y 0 0 to be4.408
x0 the positive at themtop s of the 4.41mEmpire s .State
2 x x0 2 15.0 m 2
Building. The initial velocity is v0 0 , and the acceleration is a 9.80 m s .
elapsed time
24. The sprinter startsisfromfound by solving
rest. The average Eq. 2-11a for time.is found from Eq. 2-11c.
acceleration
(a) The elapsed time can vbe found v0 11.5 from2mEq. s 2-11b, with x replaced
2 0 2 by y.
vv 2 vv0 2 at2 a x tx v v0 11.5
2.61 ms s 0
a 4.408 m 2 y 2 2 380 m 4.408 m s 2 4.41m s 2 .
y0 y0 v0 t 012 at 2 a t2 x x0s 2 15.0 m 8.806 s 8.8 s .
a 9.80 m s 2
The elapsed time is found by solving Eq. 2-11a for time.
25. (b)TheThewords final“slowing
velocitydown uniformly”
can vbe found from implies that the car has a constant acceleration. The distance
v0 11.5 mequation
s 0 (2-11a).
of travelv is vv0 found
v0 form
at at t0combining
9.80 m Eqs. s 2-7
2
8.806and2s 2-8.2.61 86 ms s
v0 v 21.0a m s 4.408 0 m sm s
x x0 t 6.00 sec 63.0 m .
2 2
36.
4. AChoose
25. The words
stone upward“slowing
is thrown to be the positive
down
vertically uniformly”direction,
upward withand
implies taketheyof
that
a speed 0car
0 toabe
has
18.0m/s. at
constant
How the height is where
acceleration.
fast it moving the
Theball wasit reaches
distance
when
of travel is found form combining Eqs. 2-7 and 2-8.
26. ahit.height of 11.0m
upwardofand thehow v0 islong
22 mis The
srequired 0 atto reach isthis
theheight? Why are therem vstwo .0 answers?
2
The For
finalthe velocity path, car zero. , v initial the
velocitytop of found path,from andEq. a 2-11c 9.80
with and
v0 v 21.0 m s 0 m s
(a) The
solving x fordisplacement
x0 v0 . t can be found from Eq. 6.00 2-11c, secwith x63.0 replaced
m . by y . [3 marks]
2 2 2 2 2

v 2 vv202 2v 2a x2 a x0y y v0 y v 2 y 2a v x vx00 0 00 2 227.00 m sm s 2 92 m


25 m 36 m s
0 0 0 2
2 a
26. The final velocity of the car is zero. The initial velocity is found from Eq. 2-11c with v 0 and 2 9.80 m s
solving
© 2005(b) The
Pearson for
time v0 .ofInc.,
Education, flight
Uppercan be River,
Saddle found NJ.from Eq.reserved.
All rights 2-11b, This withmaterial
x replaced by under
is protected y , using a displacement
all copyright laws as they of 0
currently exist.
for2Notheportion of this materialof
displacement may
thebeball
reproduced,
returningin any toform theor by any means,
height from without
which permission
it was in writing from the
hit.
publisher. v v02 2 a x x0 v0 v 2 2a x x0 0 2 7.00 m s 2 92 2 m22 m s36 m s
1 2 19
1
2 v0
y y0 v0 t 2 at 0 t v0 2 at 0 t 0,t 4.5 s
a 9.80 m s 2
© 2005 Pearson Education, Inc., Upper Saddle River, NJ. All rights reserved. This material is protected under all copyright laws as they
The
currently exist. No result ofthis
portion of t =material
0 s is may
the betime for thein original
reproduced, any form ordisplacement
by any means, without of zero (when
permission the ball
in writing fromwas
the hit), and
publisher. the result of t = 4.5 s is the time to return to the original displacement. Thus the answer is t =
4.5 seconds. 19

37. Choose upward to be the positive direction, and take y 0 0 to be the height from which the ball

was thrown. The acceleration is a 9.80 m s 2 . The displacement upon catching the ball is 0,
27 it was thrown. The starting speed can be
assuming it was caught at the same height from which
found from Eq. 2-11b, with x replaced by y.
the final velocity, when the ball is moving downwards. Both of these velocities have the same
Chapter 2
magnitude, and so the ball has the same speed at the end of Describing
its flight asMotion: Kinematics in One Dimension
at the beginning.

42. Choose upward to be the positive direction, and y 0 0 to be the height from which the stone is
H
to give tWe
thrown. 1
Thave v0, where T s ,3.2
18.0 m a s is themtotal
9.80 s 2 ,time
and elapsed
y y0 from
11.0 dropping
m. the rock to hearing the
vs
(a) The velocity can be found from Eq, 2-11c, with x replaced by y.
sound. Insert
v 2 vthis
2
0
2 a y y0 for0 t1 into the equation for H, and solve for H.
expression
2
2 H g m 2s 2 gT
H v 12 g Tv0 2 ay 18.0 H 2 9.80
1 H m s 21 gT
11.0
2 m
0 10.4 m s
2 2
vs 2v s
vs
Thus the speed
5
is2 v 10.4 m s
4.239 10 H 1.092 H 50.18 0 H 46.0 m, 2.57 10 4 m
(b) The time to reach that height can be found from equation (2-11b).
H
If the larger answer is used
1 2 in t1 T2 2 18.0 m s
, a negative 2time11.0 m results, and so the physically
of fall
y y0 v0 t 2 at t vs9.80 m s 2 t 0
9.80 m s 2
correct answer is tH 2.245
t 2 3.673 46 m 0. t 2.90 s , 0.775 s
(c) There are two times at which the object reaches that height – once on the way up t 0.775 s ,
46. Choose upward to be the positive direction, and y 0 0 to be the location of the nozzle. The
and once on the way down t 2.90 s .
initial velocity is v0 , the acceleration is a 9.8 m s 2 , the final location is y 1.5 m , and
43. the
5. AThe timeisof(100
10-cm
stone flight
thrownmm)is t 2.0
apple
vertically .diameter
Usingwith
has saupward Eq.about
of 2-11b and
6 mm
a speed assubstituting
of measured from
12.0m/s y for
in the thex edge
givesof
photograph. the following.
Thus any70.0m high.
a cliff
distances measured from the picture need to1 be2multiplied by 100
1 / 6. Choose
2 the downward
2
How long does it take 2to reach the y bottom
at of1.5 themcliff2 and9.8what
m s is its
2.0speed
s before hitting? What
direction v0 t 12 atChoose
y toy0be positive. vy00 0 to2be stem of the apple on the THIRD image from 9.1m s of
the top
was the total distance that it traveled? t Ignore air resistance. 2.0 s
the picture. It is the first picture in which the stem of the apple is visible. The velocity of the apple
[3 marks]
at that position is not 0, but it is not known either. Call it v0 . We will choose that the time at that
47. Choose downward to be the positive direction, and y 0 0 to be at the top of the cliff. The initial
point is t 0 , and we call the time interval from one picture to the next to be T . The acceleration
2
velocity
of is vis0 a 12.0
the apple g 9.8 mm s 2 . acceleration is a 9.80 m s , and the final location is y 70.0 m .
s , the
(a) Using Eq. 2-11b and substituting y for x, we have
y y0 v0 t 12 at 2 4.9 m s 2 t 2 12.0 m s t 70 m 0 t 2.749 s , 5.198 s .
© 2005 Pearson The positive
Education, answer
Inc., Upper is River,
Saddle the physical answer:
NJ. All rights t material
reserved. This .
5.20 sis protected under all copyright laws as they
currently exist. No portion of this material may be reproduced, in any form or by any means, without permission in writing from the
publisher.
(b) Using Eq. 2-11a, we have v v at 0
12.0 m s 9.80 m s 2 5.198 s 38.9 m s .
24
(c) The total distance traveled will be the distance up plus the distance down. The distance down
will be 70 m more than the distance up. To find the distance up, use the fact that the speed at
the top of the path will be 0. Then using Eq. 2-11c:
2
2 2 v2 v02 0 12.0 m s
v v 0
2a y y0 y y0 0 7.35 m .
2a 2 9.80 m s 2
Thus the distance up is 7.35 m, the distance down is 77.35 m, and the total distance traveled is
84.7 m .

48. Choose upward to be the positive direction, and y 0 0 to be the level from which the ball was
thrown. The initial velocity is v0 , the instantaneous velocity is v 13 m s , the acceleration is
a 9.80 m s 2 , and the location of the window is y 28 m .
(a) Using Eq. 2-11c and substituting y for x, we have
v 2 v02 2 a y y0
2
v0 v2 2a y y0 13 m s 2 9.8 m s 2 28 m 27 m s
Choose the positive value because the initial direction is upward.

© 2005 Pearson Education, Inc., Upper Saddle River, NJ. All rights reserved. This material is protected under all copyright laws as they
currently exist. No portion of this material may be reproduced, in any form or by any means, without permission in writing from the
publisher.
26


28
Chapter 2 Describing Motion: Kinematics in One Dimension

Topic 2.1b Kinematics Problems


about t 27 s .
(c) The indication of a 0 velocity on a position-time graph is a slope of 0, which occurs at about
Calculation-based Questions
from t 38 s .
(d) graph
1. The belowmoves
The object showsin the
bothvelocity of a When
directions. theaslope
train as is positive,
function of time.from t 0 s to t 38 s ,
the object is moving in the positive direction. When the slope is negative, from t 38 s to
t 50 s , the object is moving in the negative direction.
Giancoli Physics: Principles with Applications, 6th Edition
52. Slightly different answers may be obtained since the data comes from reading the graph. We assume
that the short, nearly horizontal portions of the graph are the times that shifting is occurring, and
(b)
thoseAt the top
times are of
notitscounted
path, the
as velocity will
being “in” be 0, and
a certain so we can use the initial velocity as found
gear.
above, along with Eq. 2-11c. v2 24 m s 14 m s
(a) The average acceleration in 2nd gear is given 2 by 2 a2 2 2.5 m s 2 .
2 2 v v0 0t 27 m8 ss 4 s
v v0 2 a y y 0 y y0 0 2 37 m
2a v2 9.8
44 mm ss2 37 m s
The average acceleration in 4th gear is given by a4 4
0.64 m s 2 .
(c) a.WeAtwant
what thetime
timewas its velocity
elapsed the greatest?
from throwing (speed v0 27t4m s ) to 27reaching
s 16 s the window (speed
(b) b.The
v During
m swhich
distance
13 periods,
). traveled
Using beif determined
can2-11a,
Eq. any,
wewashave:the velocity
from constant?
a velocity-time graph by calculating the area
c. During what periods, ifv any,
between the graph and the v v0 was
0 axis,
13 mthes acceleration
bounded by
27 m s the constant?consideration. For this case,
times under
v approximate
d.weWhen
will vwas
0
atthe magnitude
thet area asof the acceleration
a rectangle. 1.4greatest?
the s .
v f v0 a44 m s 37 9.80 m s 2
height v elapsed from the window (speed
m s
40.5 s mwidth t 27the s street
16 s (speed
11 s [4 marks]
(d) We want the time v0m 13 s ) to reaching
2 2
Esimtate
v 27 the
m s distance the object travelled
Thus the distance traveled is d v thave:40.5 m s 11 s
). Using Eq. 2-11a, we 450 m .
e. during the first minutev and; v0 27 m s 13 m s
v v0 at t 4.1 s .
f. indifferent
53. Slightly the secondanswersminutemay beaobtained since 9.80the 2
m sdata comes from reading the graph. We assume
that the short, nearly horizontal portions of the graph are the times that shifting is occurring, and [2 marks]
those times
49. Slightly are notanswers
different countedmay as being “in” a certain
be obtained gear.
since the data comes from reading the graph.
v 14 m s 0 m s
(a) The greatest
average velocity
acceleration is found at the
in first gearhighest by aon the graph, which is at t 4 48
point
is given m s2. .
t
(b) The indication of a constant velocity on a velocity-time graph is4as slope
0s
of 0, which occurs from
v 37 m s 24 m s
(b) The average
t 90 s to acceleration
t 108 s . in third gear is given by a 3 m s2 .
t 14 s 9 s
(c) The indication of a constant acceleration on a velocity-time graph is a constant slope, which
v 52 m s 44 m s 2
(c) occurs
The average
from acceleration
t 0 s to tin fifth 38 s gear is given
, again from byt a65 s to t 83 s , and again0.35 fromm s .
t 50 s 27 s
(d) Thet 90 s to acceleration
average t 108 s . through the first four gears is given by
(d) The magnitude v of 44 the
m sacceleration
0m s is greatest when the magnitude of the slope is greatest, which
a 1.6 m s 2 .
occurs fromt t 65 27s s to 0ts 83 s .

54. Slightly different


50. Slightly different answers
answers may
may be
be obtained
obtained since
since the
the data
data comes
comes from
from reading
reading thethe graph.
graph.
(a)
(a) The instantaneous velocity is given by the slope of the tangent line to the curve.find
To estimate the distance the object traveled during the first minute, we need to At the
t area
10.0 s ,
under the graph, from t = 0 s to t = 603s.m Each0 "block" of the graph represents an "area" of
thex slope is approximately v 10 0.3 m s .
10 m s 10 s 100 m . By counting and estimating, there are about 17.5 blocks under
10.0 s 0
the 1st minute of the graph, and so the
(b) At t 30.0 s , the slope of the tangent line distance
to thetraveled during
curve, and thusthe
the1stinstantaneous
minute is about
velocity, m .
1750 is
(b) For the second minute, there22 mare8about
m 5 blocks under the graph, and so the distance traveled
approximately v 30 1.4 m s .
during the second minute35 is sabout
25 s500 m .
Alternatively, average accelerations x 5 m x 0 m 1.5 m 0
(c) The average velocity is given bycan v be estimated for various portions of .30the
m graph,
s . and then the
uniform acceleration equations may be applied. 5.0 For
s 0instance,
s for5.0
part
s (a), break the motion up into
x 30
© 2005 Pearson Education, Inc., Upper Saddle River, NJ. All rights m This
reserved. x 25 m is protected
material 16 m under
9 mall copyright laws as they
(d)exist.
currently TheNoaverage
portion of velocity
this materialismay
given by v in any form or by any means, without permission in 1.4
be reproduced, writing s . the
m from
publisher. 30.0 s 25.0 s 5.0 s
x 5028m x 40 m 10 m 19.5 m
(e) The average velocity is given by v 0.95 m s .
50.0 s 40.0 s 10.0 s
51. Slightly different answers may be obtained since the data comes from reading the graph.
29
(a) The indication of a constant velocity on a position-time graph is a constant slope, which occurs
from t 0 s to t 18 s .
Giancoli Physics: Principles with Applications, 6th Edition

(b) At the top of its path, the velocity will be 0, and so we can use the initial velocity as found
above, along with Eq. 2-11c.
2
2 2 v2 v02 0 27 m s
v
2. The position ofva rabbit
2 a y y0 y y0
along a straight
0
0
tunnel2 aas a function
2 9.8of m
time 37 m
s 2 is plotted below. What is the
instantaneous velocity
(c) We want the time elapsed from throwing (speed v0 27 m s ) to reaching the window (speed
a. at t = 10.0s and,
b. v at13
t =m30.0s?
s ). Using Eq. 2-11a, we have:
v v0 13 m s 27 m s [2 marks]
v v0 at t 1.4 s .
a 9.80 m s 2
(d) We want the time elapsed from the window (speed v0 13 m s ) to reaching the street (speed
v 27 m s ). Using Eq. 2-11a, we have:
v v0 27 m s 13 m s
v v0 at t 4.1 s .
a 9.80 m s 2

49. Slightly different answers may be obtained since the data comes from reading the graph.
(a) The greatest velocity is found at the highest point on the graph, which is at t 48 s .
(b) The indication of a constant velocity on a velocity-time graph is a slope of 0, which occurs from
t 90 s to t 108 s .
(c) The indication of a constant acceleration on a velocity-time graph is a constant slope, which
Whatoccurs
is the from
averaget velocity
0 s to t 38 s , again from t 65 s to t 83 s , and again from
c. t between
90 s to tt= 0108
ands t. = 5.0s,
(d)d.Thebetween t = of
magnitude 25.0s and t = 30.0s
the acceleration and, when the magnitude of the slope is greatest, which
is greatest
e.occurs
between t = 40.0s and t
from t 65 s to t 83 s . = 50.0s?
[3 marks]
50. Slightly different answers may be obtained since the data comes from reading the graph.
(a) The instantaneous velocity is given by the slope of the tangent line to the curve. At t 10.0 s ,
3m 0
the slope is approximately v 10 0.3 m s .
10.0 s 0
(b) At t 30.0 s , the slope of the tangent line to the curve, and thus the instantaneous velocity, is
22 m 8 m
approximately v 30 1.4 m s .
35 s 25 s
x 5 m x 0 m 1.5 m 0
(c) The average velocity is given by v .30 m s .
5.0 s 0 s 5.0 s
x 30 m x 25 m 16 m 9 m
(d) The average velocity is given by v 1.4 m s .
30.0 s 25.0 s 5.0 s
x 50 m x 40 m 10 m 19.5 m
(e) The average velocity is given by v 0.95 m s .
50.0 s 40.0 s 10.0 s

51. Slightly different answers may be obtained since the data comes from reading the graph.
(a) The indication of a constant velocity on a position-time graph is a constant slope, which occurs
from t 0 s to t 18 s .
(b) The greatest velocity will occur when the slope is the highest positive value, which occurs at
© 2005 Pearson Education, Inc., Upper Saddle River, NJ. All rights reserved. This material is protected under all copyright laws as they
currently exist. No portion of this material may be reproduced, in any form or by any means, without permission in writing from the
publisher.
27


30
3. A certain type of automobile can accelerate approximately as shown in the velocity – time graph as
shown2below. (The short flat spots in the curve represent
Chapter shifting
Describing of theKinematics
Motion: gears.) in One Dimension

about t 27 s .
(c) The indication of a 0 velocity on a position-time graph is a slope of 0, which occurs at about
from t 38 s .
(d) The object moves in both directions. When the slope is positive, from t 0 s to t 38 s ,
the object is moving in the positive direction. When the slope is negative, from t 38 s to
t 50 s , the object is moving in the negative direction.

52. Slightly different answers may be obtained since the data comes from reading the graph. We assume
that the short, nearly horizontal portions of the graph are the times that shifting is occurring, and
those times are not counted as being “in” a certain gear.
v2 24 m s 14 m s
(a) The average acceleration in 2nd gear is given by a2 2.5 m s 2 .
t2 8s 4s
v4 44 m s 37 m s
The average acceleration in 4th gear is given by a4 0.64 m s 2 .
t4 27 s 16 s
(b) The distance traveled can be determined from a velocity-time graph by calculating the area
Estimatebetween
the average acceleration
the graph and the v when it isbounded
0 axis, in by the times under consideration. For this case,
we will approximate the area as a rectangle.
a. first,
b. third, height v v f v0 44 m s 37 m s 40.5 m s width t 27 s 16 s 11 s
c. fifth gear. 2 2
d. What is itsdistance
Thus the average acceleration
traveled is d vthrough
t themfirst
40.5 s 11four
s gears?
450 m .
[4 marks]
53. Slightly different answers may be obtained since the data comes from reading the graph. We assume
that the short, nearly horizontal portions of the graph are the times that shifting is occurring, and
those times are not counted as being “in” a certain gear.
v 14 m s 0 m s
(a) The average acceleration in first gear is given by a 4 m s2 .
t 4s 0s
v 37 m s 24 m s
(b) The average acceleration in third gear is given by a 3 m s2 .
t 14 s 9 s
v 52 m s 44 m s
(c) The average acceleration in fifth gear is given by a 0.35 m s 2 .
t 50 s 27 s
(d) The average acceleration through the first four gears is given by
v 44 m s 0 m s
a 1.6 m s 2 .
t 27 s 0 s

54. Slightly different answers may be obtained since the data comes from reading the graph.
(a) To estimate the distance the object traveled during the first minute, we need to find the area
under the graph, from t = 0 s to t = 60 s. Each "block" of the graph represents an "area" of
x 10 m s 10 s 100 m . By counting and estimating, there are about 17.5 blocks under
the 1st minute of the graph, and so the distance traveled during the 1st minute is about 1750 m .
(b) For the second minute, there are about 5 blocks under the graph, and so the distance traveled
during the second minute is about 500 m .

Alternatively, average accelerations can be estimated for various portions of the graph, and then the
31For instance, for part (a), break the motion up into
uniform acceleration equations may be applied.
© 2005 Pearson Education, Inc., Upper Saddle River, NJ. All rights reserved. This material is protected under all copyright laws as they
Graph of the velocity Graph of the acceleration
Giancoli Physics: Principles with Applications, 6th Edition
6
30
5

20Topic 2.1c Kinematics Problems


25
4

a (m/s 2)
Graph of the velocity Graph of the acceleration
v (m/s)

15 3
6
30
10 2
5
25
5 1
Calculation-based Questions 4

a (m/s 2)
20
0 0
v (m/s)

15 0 1 2 3 4 5 6 3 0 1 2 3 4 5 6
1. A car
accelerates
10 from
t (s)13m/s
to 25m/s in 6.0s. What was its acceleration?
t (s) How far did it travel in
2

this5time? Assume constant acceleration. 1


0
0 1 2 3 4 5 6
0
[2 marks]
0 1 2 3 4 5 6
v v0 25 m s 13 m s 2
21. By2definition, the
Chapter acceleration
t (s) is a 2.0 .
m stKinematics
(s)
t 6.0 Describing
s Motion: in One Dimension
The distance of travel can be found from Eq. 2-11b.
2
33. Choose x downward
x0 v0 t to 1
at 2 13 m s 6.0
2 be the positive direction,
1
v sv0 2 25 2.0mm
and
2
s s 13
take 6.0s0s at the114
y 0m topm of the cliff. The initial
2
21. By definition, the acceleration is a 2.0 m s .
velocity is v0 0 , and the acceleration is t a 9.80 m 6.0s s2 . The displacement is found from equation
2. A(2-11b),
22. cardistance
The
The slows
withdown
acceleration of from
can
travel
x replaced
be
can 23m/s
found
bybe to rest
from
y. found from in a distance
Eq. (2-11c).
Eq. 2-11b. of 85m. What was the acceleration, assumed
constant? 2 21 2
1 2
v v 2 2.00m 2s23 2
m6.0s s 22 114 m 2
yvx2 xy0v002 vv020tta 2x12 at
atx2 0 13 m y as 0 6.0
0 s 12 09.80 m s 3.25 s 3.1my s 51.8
. m
2 x x0 2 85 m [1 mark]
22. The acceleration can be found from Eq. (2-11c).
34. v0 0 , the final velocity is
23. Choose
Assumedownwardthat the plane to bestarts
the positive
from rest. direction.
The runway The initial
distance velocity
is2 found is by solving Eq. 2-11c for
2 2 v 2 v02 0 23 m s
x x0v. v0 2 a 1m x xs 0 a 3.1m s 2 .2
v 85 km h 23.61m2 sx, and 2 85 m is a 9.80 m s . The time can be
x0 the acceleration
3.6 km h v 2
v 2
33 m s
2
0
2 2
found vby solving
v0 2 aEq. x 2-11a
x0 for thex time. x0 0
2
1.8 10 2 m
23. Assume that the plane starts from rest. The runway 2a distance
2 3.0 m iss found by solving Eq. 2-11c for
3. Estimate
x xv0 . vhow atlong it ttook King
v v0 Kong 23.61m to falls straight
0
2.4 down
s from the top of the Empire State Building
24. (380m high) starts
and his velocity a just before
9.80 m2he s touched the ground. Ignore air resistance.
0 2
The sprinter from rest. The average vaccelerationv02 33ismfound
s
2
from
0 Eq. 2-11c.
v 2 2
v0 2 a x x0 x x0 2 2 1.8 10 2 m [2 marks]
2 2 v v022 a 11.5 2 m
3.0 sm s 20 2 2
35. Choose v downward
v0 2 a to x be a direction, and take y 0 0 to be4.408
x0 the positive at themtop s of the 4.41mEmpire s .State
2 x x0 2 15.0 m 2
Building. The initial velocity is v0 0 , and the acceleration is a 9.80 m s .
elapsed time
24. The sprinter startsisfromfound by solving
rest. The average Eq. 2-11a for time.is found from Eq. 2-11c.
acceleration
(a) The elapsed time can vbe found v0 11.5 from2mEq. s 2-11b, with x replaced
2 0 2 by y.
vv 2 vv0 2 at2 a x tx v v0 11.5
2.61 ms s 0
a 4.408 m 2 y 2 2 380 m 4.408 m s 2 4.41m s 2 .
y0 y0 v0 t 012 at 2 a t2 x x0s 2 15.0 m 8.806 s 8.8 s .
a 9.80 m s 2
The elapsed time is found by solving Eq. 2-11a for time.
25. (b)TheThewords final“slowing
velocitydown uniformly”
can vbe found from implies that the car has a constant acceleration. The distance
v0 11.5 mequation
s 0 (2-11a).
of travelv is vv0 found
v0 form
at at t0combining
9.80 m Eqs. s 2-7
2
8.806and2s 2-8.2.61 86 ms s
v0 v 21.0a m s 4.408 0 m sm s
x x0 t 6.00 sec 63.0 m .
2 2
36.
4. AChoose
25. The words
stone upward“slowing
is thrown to be the positive
down
vertically uniformly”direction,
upward withand
implies taketheyof
that
a speed 0car
0 toabe
has
18.0m/s. at
constant
How the height is where
acceleration.
fast it moving the
Theball wasit reaches
distance
when
of travel is found form combining Eqs. 2-7 and 2-8.
26. ahit.height of 11.0m
upwardofand thehow v0 islong
22 mis The
srequired 0 atto reach isthis
theheight? Why are therem vstwo .0 answers?
2
The For
finalthe velocity path, car zero. , v initial the
velocitytop of found path,from andEq. a 2-11c 9.80
with and
v0 v 21.0 m s 0 m s
(a) The
solving x fordisplacement
x0 v0 . t can be found from Eq. 6.00 2-11c, secwith x63.0 replaced
m . by y . [3 marks]
2 2 2 2 2

v 2 vv202 2v 2a x2 a x0y y v0 y v 2 y 2a v x vx00 0 00 2 227.00 m sm s 2 92 m


25 m 36 m s
0 0 0 2
2 a
26. The final velocity of the car is zero. The initial velocity is found from Eq. 2-11c with v 0 and 2 9.80 m s
solving
© 2005(b) The
Pearson for
time v0 .ofInc.,
Education, flight
Uppercan be River,
Saddle found NJ.from Eq.reserved.
All rights 2-11b, This withmaterial
x replaced by under
is protected y , using a displacement
all copyright laws as they of 0
currently exist.
for2Notheportion of this materialof
displacement may
thebeball
reproduced,
returningin any toform theor by any means,
height from without
which permission
it was in writing from the
hit.
publisher. v v02 2 a x x0 v0 v 2 2a x x0 0 2 7.00 m s 2 92 2 m22 m s36 m s
1 2 19
1
2 v0
y y0 v0 t 2 at 0 t v0 2 at 0 t 0,t 4.5 s
a 9.80 m s 2
© 2005 Pearson Education, Inc., Upper Saddle River, NJ. All rights reserved. This material is protected under all copyright laws as they
The
currently exist. No result ofthis
portion of t =material
0 s is may
the betime for thein original
reproduced, any form ordisplacement
by any means, without of zero (when
permission the ball
in writing fromwas
the hit), and
publisher. the result of t = 4.5 s is the time to return to the original displacement. Thus the answer is t =
4.5 seconds. 19

37. Choose upward to be the positive direction, and take y 0 0 to be the height from which the ball

was thrown. The acceleration is a 9.80 m s 2 . The displacement upon catching the ball is 0,
32 it was thrown. The starting speed can be
assuming it was caught at the same height from which
found from Eq. 2-11b, with x replaced by y.
the final velocity, when the ball is moving downwards. Both of these velocities have the same
Chapter 2
magnitude, and so the ball has the same speed at the end of Describing
its flight asMotion: Kinematics in One Dimension
at the beginning.

42. Choose upward to be the positive direction, and y 0 0 to be the height from which the stone is
H
to give tWe
thrown. 1
Thave v0, where T s ,3.2
18.0 m a s is themtotal
9.80 s 2 ,time
and elapsed
y y0 from
11.0 dropping
m. the rock to hearing the
vs
(a) The velocity can be found from Eq, 2-11c, with x replaced by y.
sound. Insert
v 2 vthis
2
0
2 a y y0 for0 t1 into the equation for H, and solve for H.
expression
2
2 H g m 2s 2 gT
H v 12 g Tv0 2 ay 18.0 H 2 9.80
1 H m s 21 gT
11.0
2 m
0 10.4 m s
2 2
vs 2v s
vs
Thus the speed
5
is2 v 10.4 m s
4.239 10 H 1.092 H 50.18 0 H 46.0 m, 2.57 10 4 m
(b) The time to reach that height can be found from equation (2-11b).
H
If the larger answer is used
1 2 in t1 T2 2 18.0 m s
, a negative 2time11.0 m results, and so the physically
of fall
y y0 v0 t 2 at t vs9.80 m s 2 t 0
9.80 m s 2
correct answer is tH 2.245
t 2 3.673 46 m 0. t 2.90 s , 0.775 s
(c) There are two times at which the object reaches that height – once on the way up t 0.775 s ,
46. Choose upward to be the positive direction, and y 0 0 to be the location of the nozzle. The
and once on the way down t 2.90 s .
initial velocity is v0 , the acceleration is a 9.8 m s 2 , the final location is y 1.5 m , and
43. the
5. AThe timeisof(100
10-cm
stone flight
thrownmm)is t 2.0
apple
vertically .diameter
Usingwith
has saupward Eq.about
of 2-11b and
6 mm
a speed assubstituting
of measured from
12.0m/s y for
in the thex edge
givesof
photograph. the following.
Thus any70.0m high.
a cliff
distances measured from the picture need to1 be2multiplied by 100
1 / 6. Choose
2 the downward
2
How long does it take 2to reach the y bottom
at of1.5 themcliff2 and9.8what
m s is its
2.0speed
s before hitting? What
direction v0 t 12 atChoose
y toy0be positive. vy00 0 to2be stem of the apple on the THIRD image from 9.1m s of
the top
was the total distance that it traveled? t Ignore air resistance. 2.0 s
the picture. It is the first picture in which the stem of the apple is visible. The velocity of the apple
[3 marks]
at that position is not 0, but it is not known either. Call it v0 . We will choose that the time at that
47. Choose downward to be the positive direction, and y 0 0 to be at the top of the cliff. The initial
point is t 0 , and we call the time interval from one picture to the next to be T . The acceleration
2
velocity
of is vis0 a 12.0
the apple g 9.8 mm s 2 . acceleration is a 9.80 m s , and the final location is y 70.0 m .
s , the
(a) Using Eq. 2-11b and substituting y for x, we have
y y0 v0 t 12 at 2 4.9 m s 2 t 2 12.0 m s t 70 m 0 t 2.749 s , 5.198 s .
© 2005 Pearson The positive
Education, answer
Inc., Upper is River,
Saddle the physical answer:
NJ. All rights t material
reserved. This .
5.20 sis protected under all copyright laws as they
currently exist. No portion of this material may be reproduced, in any form or by any means, without permission in writing from the
publisher.
(b) Using Eq. 2-11a, we have v v at 0
12.0 m s 9.80 m s 2 5.198 s 38.9 m s .
24
(c) The total distance traveled will be the distance up plus the distance down. The distance down
will be 70 m more than the distance up. To find the distance up, use the fact that the speed at
the top of the path will be 0. Then using Eq. 2-11c:
2
2 2 v2 v02 0 12.0 m s
v v 0
2a y y0 y y0 0 7.35 m .
2a 2 9.80 m s 2
Thus the distance up is 7.35 m, the distance down is 77.35 m, and the total distance traveled is
84.7 m .

48. Choose upward to be the positive direction, and y 0 0 to be the level from which the ball was
thrown. The initial velocity is v0 , the instantaneous velocity is v 13 m s , the acceleration is
a 9.80 m s 2 , and the location of the window is y 28 m .
(a) Using Eq. 2-11c and substituting y for x, we have
v 2 v02 2 a y y0
2
v0 v2 2a y y0 13 m s 2 9.8 m s 2 28 m 27 m s
Choose the positive value because the initial direction is upward.

© 2005 Pearson Education, Inc., Upper Saddle River, NJ. All rights reserved. This material is protected under all copyright laws as they
currently exist. No portion of this material may be reproduced, in any form or by any means, without permission in writing from the
publisher.
26


33
Topic 2.1d Projectile Motion Problems
Conceptual Questions
(These questions are not in an IB style but instead designed to check your understanding of the concept of this topic. You should
try your best to appropriately communicate your answer using prose)

1. Two cannon balls A and B are fired from the ground with identical initial speeds, but with θA larger
than θB. (a) Which cannonball reaches a higher elevation? (b) which stays longer in the air? (c)
Which travels farther?

2. A projectile is launched at an angle of 30° to the horizontal with a speed of 30m/s. How does the
horizontal component of its velocity 1.0s after launch compare with its horizontal component of
velocity 2.0s after launch?

Calculation Based
3. A tiger leaps horizontally from a 6.5m high rock with a speed of 3.5m/s. How far from the base of
the rock will she land?

4. A diver running 1.8m/s dives out horizontally from the edge of a vertical cliff and 3.0s later reaches
the water below. How high was the cliff and how far from its base did the diver hit the water?

5. A football is kicked at ground level with a speed of 18.0m/s at an angle of 35.0° to the horizontal.
How much later does it hit the ground?
34
6. A projectile is fired with an initial speed of 65.2m/s at an angle of 34.5° above the horizontal on a
long flat firing range. Determine (a) the maximum height reached by the projectile, (b) the total
time in the air, (c) the total horizontal distance covered (that is the range), and (d) the velocity of
the projectile 1.50s after firing.

7. A projectile is shot from the edge of a cliff 125m above ground level with an initial speed of 65.0m/s
at an angle of 37.0° with the horizontal, as shown below. (a) Determine the time taken by the
projectile to hit point P at ground level. (b) Determine the range X of the projectile as measured
from the base of the cliff. At the instant just before the projectile hits point P, find (c) the horizontal
and the vertical components of its velocity, (d) the magnitude of the velocity, and (e) the angle
made by the velocity vector with the horizontal. (f) Find the maximum height above the cliff top
reached by the projectile.

35
36
Since the forces are parallel to each other, both pointing down the plane, they add, causing a larger
magnitude force and a larger acceleration. On the way down, those same two forces are opposite of
each other, because the force of friction is now directed up the plane. With these two forces being
opposite
Chapter 4 of each other, their net force is smaller, and so the acceleration is smaller.
Dynamics: Newton’s Laws of Motion

21. Assume your weight is W. If you weighed yourself on an inclined plane that is inclined at angle q,
3. Use Topic 2.2a Kinematics Problems
the bathroom
Newton’s scale would
second read
law to the magnitude
calculate of the normal force between you and the plane, which
the tension.
would be W cosq. 2
Chapter 4 ƒ F = FT = ma = ( 960 kg ) 1.20 m s = 1.15 ì 10 N
3
( )
Dynamics: Newton’s Laws of Motion

Calculation-based
4. In all cases,
Solutions Questions
W = mg
to Problems
3. Use Newton’s
, where g changes with location.
second law to calculate the tension.

1.
(a) WEarthF ==mFg Earth
Use ƒ
Newton’s = ma
second
= (=76( 960
law to
( ( ) )
kg ) kg 9.8 )
calculate
m1.20s 2 m= s7.4
the
2 ì 10 N
force.
2
= 1.15 ì 10 3 N 2
1. A net force of 265N accelerates a bike and rider at 2.30m/s . What is the mass of the bike and the
T

rider
(b) W ƒ
together?
F =m
Moon ma g Moon = ( 76kgkg
= ( 60.0
4. In all cases, W = mg , where g changes with location.
((
) )1.25 ))
1.7 m 2
m ss 2 == 1.3 75.0 ì 10NN
2

[1 mark]
2. Use (a) Newton’s
WEarth = msecond 76 to
((
(c) WMars = mg Mars = ( 76 kg ) 3.7 m s 22 = 2.8 ì 10 22N
g Earth = (law
))
) 9.8 m sthe =mass.
kgcalculate 7.4 ì 10 N
(d) W = m g = ( ƒ) F
76 kg ( ) 0 m s 2
265 N N = 0
(b) ƒ
Space
WMoon F ==ma mg Moon
Space
ç =m( 76 (
= kg ) 1.7
a
) = m s 2 =2 1.3
2.30 m s
= 115 ì 10kg 2
N
d
5. (a) The 20.0 kg box resting
(c) WMars = mg Mars = ( 76 kg ) 3.7 m s = 2.8 ì 10 N (on )the table 2 has the free-body 2 diagram shown. Its weight F N

2.
© 2005What
Chapter(d)
is
4 W
Pearson
mg = ( Inc., Upper
isEducation,
the=weight20.0
mg Space of
kg (
) Saddle
= (a76
9.80
76kg ( ) )
m s 2

)beastronaut
kgRiver, 0m
= 2
NJ. sAll
196 N .
= 0 reserved.
rights N
Since the box is at
This material is protected
rest, the
Dynamics:
net force
under allNewton’s
on
copyright lawsLaws of Motion
as they
Space
a.theon
currently exist. No Earth
portion of this material may reproduced,
box must be 0, and so the normal force must also be 196 N .
in any form or by any means, without permission in writing from the
publisher. d d
b. on the Moon (g = 1.7m/s2) 69free-body d
F mg
5.
3. (b) (a) The
Usec.Free-body
Newton’s 20.0 kg box
diagrams
second resting
laware on
toshown the
calculate table has the
for both boxes. F12 is the force on box 1 (the diagram shown. Its weight
) 2 the tension.
2 N
on Mars (g = 3.7m/s dforce on
is
d.top
mg
ƒinbox) =
Fouter
=F ( =space
dueT
20.0 toma kg
box (
=)traveling
9.80
(2960
(thekg m
()
s
bottom
) with =
1.20box),
196
) N . Since
m sand=isvelocity?
constant
2 the
the ìnormal
1.15 box is at rest,
10 N force on box 1. F21
3 the net d
F =
d
F
d N1 12
isthetheboxforcemust onbe box0, 2and duesotothe boxnormal1, andforce has the mustsame be 196 Nas. F12 by
alsomagnitude [4 marks]
d Top box (#1) d
4. In all cases, W = mg , d
where g changes with location. mg
(b) Newton’s
Free-body3diagrams rd
law. FN2areisshown the force for of both table on F
theboxes. 12 is
box 2.theThatforce on box
is the normal1 (the
d dm gd d
(a) force
WEarth
top =m
on
box) box g Earth
due box( 762 kg
2.to =Since both
(the (
) bottom
9.8
boxes )
m sare 2
box), = 7.4
andì the
at rest, 10the
is
2
N normal
net force on forceeach onboxboxmust
1. F21 FN11= F12
be 0. Write Newton’s 2nd law in the2 vertical direction for each box, dtaking the
is
(b) upward the force
WMoon mg on
=direction box
Moon to
= ( 762bedue
d
(
kg )to1.7
positive. )
boxm1,sand=has 1.3the ì 10same2
N magnitude as F12 by
d Top box (#1)
rd
(c) WMarsƒ= m1g Mars N1= ( 761 N2kg ) 3.7 m s = 2.8 ì 10 N
Newton’s F 3 = Flaw. - m F g =is 0 the force of the table on box 2. That is the normal FN2
( ) 2 2

m1g
d
force on box 2. Since both boxes are2 at rest, the net force on each box must
(d) be WSpace
0. N1= mg Space
Write = ( 76 kg
Newton’s ((
F = m1 g = (10.0 kgnd) 9.80 m2 s = 98.0 N = F12 = F21
2 ) law ) )
0 minsthe=vertical 0 N direction for each box, taking the (#2)
Bottom box

upward ƒ Fdirection
2
= FN 2 -toFbe 21
-positive.
m2 g = 0
d d dd
5. (a) The 20.0 ƒ F kg = boxF - m
resting g = on 0 the table has the free-body diagram shown. Its weight FmN22 g F FN21
3. What is the FN 2average
=1F21 + mforce
N1
2
1
g = 98.0required N + ( 20.0 to stop (
kg ) a9.80 )
1100-kg
m s car 2
in 8.0s
= 294 N if the car is travelling at 95km/h?
is mgF= (=20.0
N1
m1 g kg (
= ()10.0 9.80 kgm ( )
) 9.80
2
s =m196 ) s2 N = 98.0 . Since N =theF12box = Fis21 at rest, the net force on Bottom box [3 marks]
(#2)
6. Find the the average F 2 acceleration F21 -sofrom Eq.0 2-2.force The must average alsoforce on the N .car is found from Newton’s
second law.
ƒ
box must = beFN 0,
2
- and mthe2
g normal
=
d
be 196
d d d
(b) Free-body F m g Fmg
FN 2 =diagrams
F21 + m2 gare shown Nm+for both boxes. is the force on box 1 (the 2

v = 0 v0 = ( 95 km h ) ∆
=≈98.00.278 (s20.0
=
(
’ kg ) 9.80 m 12s =v 294
26.4 m s
) a
2

=
- v0 N 0 - 26.4 m ds
= = - 3.30 m s 2
21

÷ d d
top box) due to box 2 (the1km
« bottom h box), and is the normal force on box8.0 1.s F21
avg
◊ t F = F
6. Find is thetheaverage acceleration d found from Newton’s 12 N1
force on box 2 due from to boxEq. 1, 22-2.
and has Thethe average
same force on theascar
magnitude F12isby
second Favglaw.
rd
(
= maavg = (1100 dkg ) - 3.3 m s = - 3.6 ì 10 N ) 3
Top box (#1)
Newton’s 3
The negative sign indicatesN2the law. F is
≈0.278the force
direction of the table on
m s ’of the force, in the opposite box 2. That is the
v - v0 direction normal
0 - 26.4 to m thes initial velocity.
v = 0 on vbox
force 0
= (2.95Since
km h )both ∆ 1km boxes ÷
are =at26.4rest, mthe s netaavg force= on each = box must = - 3.30 m s 2m gd
h ◊ t 8.0 s 1
7. The average be 0. Write forceNewton’son the pellet 2«nd is law itsinmassthe vertical
times itsdirection
average for each box, taking
acceleration. The averagethe acceleration is
4. What
foundupwardis the
Ffrom average
ma
= Eq. direction = (1100
2-11c. force
to be
For kg
the(needed - 3.3 mto
positive.
) pellet, )
v0 s= accelerate
2
0=, -v 3.6 = 125ì 10am37.00g
Ns , andpellet x - x0from restmto. 125m/s over a distance
= 0.800
avg avg d
ofThe
0.800m
negative ƒalong
F2sign =the FN1 barrel
- m1 g the
2indicates =of0direction
the
2 rifle?of the force, in the opposite direction to the initial FN2 velocity.
v - v0 1
(125 m s ) - 0 [2 marks]
aavg = = = 9770 m s2
F2N1( x=-mx1 g) = (10.0
7. The average force 0on the pellet 2 kg
0.800 (
) 9.80
m m ) s 2
= 98.0
( is its) mass times its average12 acceleration. N = F = F 21
The average (#2)
Bottom
acceleration is
box

foundFavg from ƒ F
Eq.
= maavg2 = 7.00 = (
2-11c.F N2
-
ForF
ì 10
21
-
the- 3 m kg2
g
)(
pellet, = 0
9770v 0
= 0 ), v
2
m s = 68.4 N = 125 m s , and x - x 0
= 0.800 m . d d
m2 g F21
F v =-Fv0 + m2(g125
2

= N 2 Inc.,21Upper
© 2005 PearsonaEducation,
2
= 98.0
2
m s )N-+0( 20.0 kg ) 9.80
= Saddle River, NJ. All=rights 9770
(reserved.
) 2
m s = 294 N
m s 2 This material is protected under all copyright laws as they
avg
currently exist. No portion2 ( xof-this ) 2may
x0material ( 0.800 m)
be reproduced, in any form or by any means, without permission in writing from the
publisher.
6. Find the average acceleration- 3from Eq. 2-2. The 2 average force on the car is found from Newton’s
second (
= maavg = 7.00 ì 10 kg 9770 m s70
Favglaw. )( ) = 68.4 N
≈0.278 m s ’ v - v0 0 - 26.4 m s 2
© 2005 Pearson v =Education,
0 v0 Inc., = ( 95 kmSaddle
Upper h ) ∆River, NJ. All÷rights = 26.4 m s Thisamaterial
reserved. avg
= is protected= under all copyright = - 3.30
laws asm theys
currently exist. No portion of this material may « be1km h
reproduced, ◊ in any form or by any means, t
without 8.0
permission sin writing from the
publisher.
( 70 ì 103 N
Favg = maavg = (1100 kg ) - 3.3 m s 2 = - 3.6
37 )
The negative sign indicates the direction of the force, in the opposite direction to the initial velocity.
Giancoli Physics: Principles with Applications, 6th Edition

8. We assume that the fishline is pulling vertically on the fish, and that the fish is not jerking d
the line. A free-body diagram for the fish is shown. Write Newton’s 2nd law for the fish in FT
the vertical direction, assuming that up is positive. The tension is at its maximum.
ƒ F = FT - mg = ma ç FT = m ( g + a ) ç
FT 22 N d
mg
m= = 2 2
= 1.8 kg
g +a 9.8 m s + 2.5 m s
5. A 0.140kg baseball traveling at 35.0m/s strikes the catcher’s mit, which, in bringing the ball to rest,
Thus a mass of 1.8 kg is the maximum that the fishline will support with the given
recoils backward
acceleration. Since11.0cm.
the line What
broke,was the average
the fish’s force
mass must appliedthan
be greater by the ball
1.8 kg on the
(about glove?
4 lbs).
[2 marks]
9. The problem asks for the average force on the glove, which in a direct calculation would require
knowledge about the mass of the glove and the acceleration of the glove. But no information about
the glove is given. By Newton’s 3rd law, the force exerted by the ball on the glove is equal and
opposite to the force exerted by the glove on the ball. So calculate the average force on the ball, and
then take the opposite of that result to find the average force on the glove. The average force on the
ball is its mass times its average acceleration. Use Eq. 2-11c to find the acceleration of the ball, with
v = 0 , v0 = 35.0 m s , and x - x0 = 0.110 m . The initial direction of the ball is the positive direction.
2
v 2 - v02 0 - ( 35.0 m s )
aavg = = = - 5568 m s 2
2 ( x - x0 ) 2 ( 0.110 m )

(
Favg = maavg = ( 0.140 kg ) - 5568 m s 2 = - 7.80 ì 10 2 N )
Thus the average force on the glove was 780 N, in the direction of the initial velocity of the ball.

10. Choose up to be the positive direction. Write Newton’s 2nd law for the vertical d
FT
direction, and solve for the tension force.
ƒ F = FT - mg = ma ç FT = m ( g + a )
d
mg
(
FT = (1200 kg ) 9.80 m s 2 + 0.80 m s 2 = 1.3 ì 104 N )
11. Use Eq. 2-11b with v0 = 0 to find the acceleration.
2 ( x - x0 ) 2 ( 402 m ) ≈ 1 "g" ’
x - x0 = v0 t + 12 at 2 ç a= 2
= 2
= 19.6 m s 2 ∆ 2 ÷ = 2.00 g 's
t ( 6.40 s ) «9.80 m s ◊
nd
The accelerating force is found by Newton’s 2 law.
( )
F = ma = ( 485 kg ) 19.6 m s 2 = 9.51 ì 103 N

12. Choose up to be the positive direction. Write Newton’s 2nd law for the vertical direction,
d
and solve for the acceleration. FT
ƒ F = FT - mg = ma
a=
FT - mg
=
163 N - (12.0 kg ) 9.80 m s 2 ( ) = 3.8 m s 2
d
m 12.0 kg mg

Since the acceleration is positive, the bucket has an upward acceleration.

© 2005 Pearson Education, Inc., Upper Saddle River, NJ. All rights reserved. This material is protected under all copyright laws as they
currently exist. No portion of this material may be reproduced, in any form or by any means, without permission in writing from the
publisher.
71

38
Topic 2.2b Kinematics Problems

Conceptual Questions
(These questions are not in an IB style but instead designed to check your understanding of the concept of this topic. You should
try your best to appropriately communicate your answer using prose)

1. Why does a child, sat in a toy wagon, seem to fall backward when you give the wagon a sharp pull
forward?

The child has inertia – the property of an object to resist change. As you pull the wagon forward,
only the feet of the child are pulled forward and the rest of the body wants to stay in place. Hence,
the child seem to fall backward.

2. If the acceleration of an object is zero, are no forces acting on it? If only one force acts on the
object, can the object have zero acceleration? Can it have zero velocity? Explain.

The net force has to be zero if acceleration is zero. This does not mean that there are no forces
acting on it, just that all forces acting on the object are balanced (cancel each other out). If only one
force acts on the objects, this is the net force and the object accelerates.
The velocity can be zero even if the net force (and hence, the acceleration) is not zero. However, this
velocity has to change in the next instant.

3. If you walk along a log floating on a lake, why does the log move in the opposite direction?

The log is pushing you forward, but according to N3 you are also pushing the log backwards (in the
opposite direction)

39
Topic 2.2c Kinematics Problems

Calculation-based Questions
Chapter 4 Dynamics: Newton’s Laws of Motion

1. Arlene is to walk across a “high-wire” strung horizontally between two buildings 10.0m apart. The
21. sag
(a) inJust
theasrope when
the ball she ishit,
is being at ifthe
wemid-point is 10.0° asthere
ignore air resistance, shown. If her mass is
are two d 50.0kg, what is the
Fbat
tensionmaininforces
the rope at this
on the ball –point?
the weight of the ball, and the force of the bat
d
Giancoli on the ball. Physics: Principles with Applications, 6th Edition
mg[2 marks]
(b) As the ball flies toward the outfield, the only force on it is its weight, if mg
d
air resistance is ignored.
17. (a) There will be two forces on the skydivers – their combined weight, and the d
d d FA
22. The two forces
upward must
force ofbe
airoriented so that
resistance, FA .the northerly
Choose up tocomponent of thedirection.
be the positive first Write
d F1
force Newton’s
is exactly 2equal F
nd
lawtofor
thethe
southerly
skydivers.component of the second force. Thus 2

the second force must act southwesterly. See the diagram.


ƒ F = FA - mg = ma ç 0.25mg - mg = ma ç d d
F1 + F2
(
a = - 0.75 g = - 0.75 9.8 m s = - 7.4 m s 2
)
23. Consider the point in the rope directly below Arlene. That point can
2

d d
FT d
Due to the sign of the result, the direction of the
be analyzed as having three forces on it – Arlene’s weight, the acceleration is down. o
FT mg
o
(b) If they 10 10
tension in theare descending
rope towards theat constant speed,
right point then the netand
of connection, forcetheon them must d
be in
zero, m g
tension the and
ropesotowards
the force
theofleft
airpoint
resistance must be equal
of connection. to their weight.
Assuming
the rope is Fmassless,
A
= mg =those
(132 two (
kg ) tensions
9.80 m swill=be1.29
2
)
of the
ì 10same
3
N
magnitude. Since the point is not accelerating the sum of the forces must be zero. In particular,
consider the sum of the vertical forces on that point, with UP as the positive direction.
18. (a) Use Eq. 2-11c to find the speed of the person just before striking the ground. Take down to be
ƒ F = FT sin10.0o + FT sin10.0o - mg = 0 ç
the positive direction. For the person, v0 = 0 , y - y0 = 3.9 m , and a = 9.8 m s 2 .
2 mg 2 (
( 50.0 kg ) 9.80 m s 2
FT = v - v0 = o2a=( y - y0 ) ç vo= 2a (=y 1.41
)
- y0 )ì 10
= 3N ( )
2 9.8 m s 2 ( 3.9 m ) = 8.743 = 8.7 m s
2 sin10.0 2 sin10.0
(b) For the deceleration, use Eq. 2-11c to find the average deceleration, choosing down to be
positive.
24. The net force in each case is found by vector addition with components. d
2. A box weighingv = 77.0N
8.743 m srestsvon= 0a table.
y - y A=rope
0.70 tied
m to
v 2 the2 box runs vertically upward F
- v = 2 a ( y - y ) ç over
1 a pulley
(a) F =0 - F = - 10.2 N F = - F =0 - 16.0 N 0 0
and a weight
Net x
is1 hung from theNetother
y
2
end.
2
Determine the force that the table exerts onq the box if the
weight - v 2
- ( 8.743 m s ) - 16.0
FNethanging on =the
) + (other
- 16.0side
) = of = -the Npulley
m sq2 =weighs d
2 2
=a =( - 10.2
0
19.0 54.6 tan - 1 = 57.5o d F
a. 30.0N 2Dy 2 ( 0.70 m ) - 10.2 Fnet 2

b.The
60.0N
The actual
average angle from
force the x-axis
is found from isNewton’s
then 237.52ndo law.
.
c. 90.0NFNet = ma 19.0 N ) ( - 54.6 at )237
2
a= F = = ( 42=kg0.703 m sm 2 s = -o 2.3 ì 103 N .
[4 marks]
The m 27.0 kg
negative sign shows that the force is in the negative direction, which is upward.
(Hint: You should sketch a free-body diagram for the general case)
(b) FNet x = F1 cos 30o = 8.83 N FNet y = F2 - F1 sin 30o = 10.9 N d
19. Free body diagrams for the box and the weight are shown below. The dd d
FF2T Fnet
d FT
tension exerts the same 2 magnitude2 of force on both objects. - 1 10.9 o FN
(a) F ( 8.83ofN )the+hanging
IfNetthe= weight (10.9 N )weight
= 14.0 N thanq the
is less = tan
weight of 51.0
=the box,
8.83 q
the objects will not move, and the tension will be the same as the d
F 14.0 N d 30o m2 g
aweight
= Netof= the hanging weight.
= 0.520 m sThe
2
atacceleration
51.0o of the box will also m1g d
be zero, m and 27.0 kg sum of the forces on it will be zero. For the box,
so the F1

FN + FT - m1 g = 0 ç FN = m1 g - FT = m1 g - m2 g = 77.0N - 30.0 N = 47.0 N


25. We draw free-body diagrams for each bucket.
(b) The same analysis as for part (a) applies here. d d
(a) Since the buckets are at rest, their acceleration is 0. Write Newton’s FT2 FT1
nd
2 law FNfor
=m g - bucket,
each
1
m2 g = 77.0 N -UP
calling 60.0
theNpositive
= 17.0 Ndirection.
(c) SinceƒtheF1hanging weight
= FT1 - mg = 0 has
ç more weight than the box on the table, the box on the table will be
lifted up off the table, and normal2force of the table on the box will be 0 N .
FT1 = mg = ( 3.2 kg ) ( 9.8 m s ) = 31 N d d d
FT1 mg mg
ƒ F2 =theFT2player
20. (a) Just before - FT1 -leaves
mg =the 0 ground
ç on a jump, the forces on the player
would be his weight and the force of the floor pushing up the player. IfTop the(# 2) Bottom (# 1)
F = F
playerT2is jumping
T1
+ mg = 2 mg = 2 ( 3.2 kg ) 9.8 m( s 2
=)63 N
straight up, then the force of the floor pushing on the
player will be straight up – a normal force. See the first diagram. In this
case,
© 2005 Pearson whileInc.,
Education, they areSaddle
Upper touching theAll
River, NJ. floor, FN >40 mgThis
rights reserved. . material is protected under all copyright laws as they
currently exist. No portion of this material may be reproduced, in any form or by any means, without permission in writing d
from the
Chapter 4 Dynamics: Newton’s Laws of Motion

d d
have the same mass m and the same acceleration a. Note that FT2R = FT2L = FT 2 by Newton’s 3rd
law.3. A window washer pulls d
herself
d
upward usingd
a bucket-pulley system as
shown. FT2 F T2 F T1

a. Sketch a free-body diagram showing the force of gravity and the


force exerted by the rope (tension).
Write a Newton’s 2nd law expression for each car.
b. How hard must she pull downward to raise herself slowly at
ƒ F1 = FTconstant
1
- FT 2 = ma ƒ F2 = FT 2 = ma
speed?
Substitute the expression for ma from the second expression into the first one.
c. If she increases this force by 15%, what will her acceleration by?
FT 1 - FT 2 = ma = FT 2 ç FT1 = 2 FT2 ç FT1 FT2 = 2
This can also be discussed in the sense that the tension between the locomotive and the first car is
Assume
pulling 2 cars, while the mass
the tension of the
between person
the cars is onlyand theone
pulling bucket
car. is 65kg.
[4 marks]
29. The window washer pulls down on the rope with her hands with a tension force FT ,
so the rope pulls up on her hands with a tension force FT . The tension in the rope is d
also applied at the other end of the rope, where it attaches to the bucket. Thus there FT d
FT
is another force FT pulling up on the bucket. The bucket-washer combination thus
has a net force of 2FT upwards. See the adjacent free-body diagram, showing only
forces on the bucket-washer combination, not forces exerted by the combination
(the pull down on the rope by the person) or internal forces (normal force of bucket
on person).
(a) Write Newton’s 2nd law in the vertical direction, with up as positive. The net
force must be zero if the bucket and washer have a constant speed. d
mg
ƒ F = FT + FT - mg = 0 ç 2 FT = mg ç
mg ( 65 kg ) ( 9.8 m s2 ) = 320 N
FT = =
2 2
(b) Now the force is increased by 15%, so FT = 320 N (1.15 ) = 368 N . Again write Newton’s 2nd
law, but with a non-zero acceleration.
ƒ F = FT + FT - mg = ma ç
a=
2 FT - mg
=
(
2 ( 368 N ) - ( 65 kg ) 9.80 m s 2 ) = 1.5 m s 2

m 65 kg

30. Since the sprinter exerts a force of 720 N on the ground at an angle d
4. The diagram below shows a block (mass mby FN
of 22o below the horizontal, A) Newton’s
on a smooth3rd lawhorizontal
the ground will exert a force
o
of 720 N on the sprinter at an angle of 22
surface, connected by a thin cord that passes over a pulley tohorizontal.
above the a A free-body
diagram for the sprinter is shown.
second block (mB), which hangs vertically. d
FP d
(a) The horizontal acceleration will be found from the net mg
22o
horizontal force. Using Newton’s 2nd law, we have the following.
a. Draw a free-body diagram for each block, showingF the force
cos 22 o
( 720 N ) cos 22o
of gravity on each, theƒ force
F x
= F cos 22 o
ma
P(tension) exerted
= x
ç a P
xby the cord,
=
m
=
65 kg
and any normal force.
= 10.27 m s 2 ö 1.0 ì 101 m s 2
b. Apply Newton’s second law to find formulas for the
acceleration of the system and for the tension in the cord.
© 2005 Pearson Education, Inc., Upper Saddle River, NJ. All rights reserved. This material is protected under all copyright laws as they
Ignore friction
currently and the
exist. No masses
portion of the
of this material may bepulley and
reproduced, in anycord.
form or by any means, without permission in writing from the
publisher.
76
[3 marks]

41
(b) Eq. 2-11a is used to find the final speed. The starting speed is 0.
v = v0 + at ç (
v = 0 + at = 10.27 m s 2 ) ( 0.32 s ) = 3.286 m s ö 3.3 m s
31. (a) See the free-body diagrams included.
y x
(b) For block 1, since there is no motion in the vertical direction, d
we have FN1 = m1 g . We write Newton’s 2nd law for the x d FT
FN1 d
direction: ƒF 1x
= FT = m1a1x . For block 2, we only need to FT
consider vertical forces: ƒF 2y
= m2 g - FT = m2 a2 y . Since the d d
m1g m2 g
two blocks are connected, the magnitudes of their accelerations
will be the same, and so let a1 x = a2 y = a . Combine the two force
equations from above, and solve for a by substitution.
FT = m1a m2 g - FT = m2 a ç m2 g - m1a = m2 a ç
m2 m1m2
m1a + m2 a = m2 g ç a=g FT = m1a = g
m1 + m2 m1 + m2

32. Consider a free-body diagram of the dice. The car is moving to the right. The d
acceleration of the dice is found from Eq. 2-11a. FT q
v - v0 28 m s - 0
v = v0 + = ax t ç ax = = = 4.67 m s 2
t 6.0 s d
Now write Newton’s 2nd law for both the vertical (y) and horizontal (x) directions. mg
mg
ƒ Fy = FT cos q - mg = 0 ç FT = cos q ƒ Fx = FT sin q = max
Substitute the expression for the tension from the y equation into the x equation.
mg
max = FT sin q = sin q = mg tan q ç ax = g tan q
cos q
ax 4.67 m s 2
q = tan - 1 = tan - 1 2
= 25o
g 9.8 m s
d d
33. (a) In the free-body diagrams below, F12 = force on block 1 exerted by block 2, F21 = force on
d d
block 2 exerted by block 1, F23 = force on block 2 exerted by block 3, and F32 = force on block
d d d
3 exerted by block 2. The magnitudes of F21 and F12 are equal, and the magnitudes of F23 and
d
F32 are equal, by Newton’s 3rd law.
d d
d FN2 FN3
d FN1
d d d d
F1 2 F21 F23 F32
F
d d d
m1g m2 g m3g

(b) All of the vertical forces on each block add up to zero, since there is no acceleration in the
vertical direction. Thus for each block, FN = mg . For the horizontal direction, we have
© 2005 Pearson Education, Inc., Upper Saddle River, NJ. All rights reserved. This material is protected under all copyright laws as they
currently exist. No portion of this material may be reproduced, in any form or by any means, without permission in writing from the
publisher.
77

42
4. The woman does work by moving the water with her hands and feet, because she must exert a force
to move the water some distance. As she stops swimming and begins to float in the current, the
current does work on her because she gains kinetic energy. Once she is floating the same speed as
Giancoli
the water, her kinetic energy does not change, and so no Physics:
net workPrinciples
is being with
doneApplications,
on her. 6th Edition

5. The kinetic force of friction opposes the relative motion between two objects. As in the example
21. The Topic 2.3a Work and Energy
superball
suggested, cannot
as the rebound
tablecloth is to a height
pulled fromgreater
under than its original
the dishes, height motion
the relative when dropped.
is for theIfdishes
it did,toit be
would violate conservation of energy. When a ball collides with the floor, the
left behind as the tablecloth is pulled, and so the kinetic friction opposes that and moves the dishesKE of the ball is in
converted into elastic PE by deforming the ball, much like compressing a spring.
the same direction as the tablecloth. This is a force that is in the direction of displacement, and Then as the ball
so
springs
positiveback
worktoisitsdone.
original
Alsoshape, that the
note that elastic PEisismoving
cloth converted to back
faster to KE.
than the Butinthat
dishes thisprocess
case, soisthat
Conceptual
“lossy”
Questions
– notisall
the friction of thenot
kinetic, elastic PE gets converted back to KE. Some of the PE is lost, primarily to
static.
friction. The superball rebounds higher than many other balls because it is less “lossy” in its
(These questions are not in an IB style but instead designed to check your understanding of the concept of this topic. You should
6. rebound
While it than many
is true other
thattrynoyour materials.
work is to
best being done on the
appropriately wall by you,
communicate yourthere is work
answer using being
prose)done inside your
arm muscles. Exerting a force via a muscle causes small continual motions in your muscles, which
22. The workand
is work, done to liftcauses
which the suitcase
you to is equal
tire. Anto the change
example in PE
of this of the suitcase,
is holding which
a heavy load at isarm’s
the weight
length.of
1. You have two springs that are identical except that spring 1 is stiffer
While at first you may hold the load steady, after a time your arm will begin to shake, which1
the suitcase times the change in height (the height of the table). than spring 2 (k > k2). On
which
(a) spring
Workthe
indicates doesis more work
NOTofdepend
motion musclesdone
on in if (a)
theyour
path, they are stretched using the same force,
as long as there are no non-conservative forces doing
arm. (b) stretched the
samework.
distance?
7. (a) In thisdoes
(b) Work case,NOT depend
the same forceon is
theapplied
time taken.
to both springs. Spring 1 will stretch less, and so more
(c) Work DOES depend
work is done on spring 2. on the height of the table – the higher the table, the more work it takes to
(b) lift the suitcase.
In this case, both springs are stretched the same distance. It takes more force to stretch spring 1,
(d) Work
and soDOES
more workdepend is on
donetheonweight
springof1.the suitcase – the more the suitcase weighs, the more
work it takes to lift the suitcase.
th
Giancoli
8. At point C the block’s speed will be less than 2vB . The Physics: same amount Principles with was
of work Applications,
done on 6theEdition
2. When
23. The power a rubber
needed bouncy-ball is dropped,
to lift the suitcase can itrequired
is the work rebound to to
liftathe height greater
suitcase, dividedthan byits
theoriginal
time thatheight?
block
Explain.
it takes. in going from A to B as from B to C since the force and the displacement are the same for each
segment. Thus the change in kinetic energy will be the same for each segment. From A to B, the
21. (a)The Since
superball work does rebound
cannot NOT depend on the greater
to a height path, the power
than will notheight
its original depend on the
when path either,
dropped. If it did, it
would assuming the time is theofsame
violate conservation energy. for all
When paths.a ball collides with the floor, the KE of the ball is
(b)
converted into elastic PE by deforming thetaken.
The power DOES depend on the time The more
ball, much time taken, the
like compressing lower the
a spring. Thenpower
as theneeded.
ball
© 2005 Pearson
(c) Education,
Theback power Inc.,
neededUpper Saddle
DOES River,
dependNJ. All rights reserved.
on the height This material
of the table. is protected
A higher under all
table copyright laws
requires as they work
more
springs to its original shape, that elastic PE is converted to back to KE. But
currently exist. No portion of this material may be reproduced, in any form or by any means, without permission in writing from the that process is
to lift
“lossy”
publisher. theall
– not suitcase. If we assume
of the elastic PE gets that the time
converted to lift
back the suitcase
to KE. Some ofisthe thePE same in both
is lost, cases, then
primarily to
to lift to the higher table takes more power. 134
friction. The superball rebounds higher than many other balls because it is less “lossy” in its
(d) The than
rebound power manyDOES depend
other on the weight of the suitcase. A heavier suitcase requires more force
materials.
to lift, and so requires more work. Thus the heavier the suitcase, the more power is needed to
22. The lift work it (in
donethetosame amount
lift the suitcaseof time).
is equal to the change in PE of the suitcase, which is the weight of
3. Why is it easier to climb a mountain
the suitcase times the change in height (the via aheight
zigzag of trail than to climb straight up?
the table).
24. The(a) climber
Work does doesNOT the same
depend amount
on theofpath, workaswhether
long asclimbing
there are straight up or via a zig-zag
no non-conservative forcespath,
doing
ignoringwork. dissipative forces. But if a longer zig-zag path is taken, it takes more time to do the work,
and
(b) soWorkthe power
does NOT output neededonfrom
depend the timethe climber
taken. is less. That will make the climb easier. It is easier
for the human body to generate
(c) Work DOES depend on the height of the table a small amount of power
– the for longthe
higher periods
table, of
thetimemorerather
workthan to to
it takes
generate a large
lift the suitcase.power for a small period of time.
(d) Work DOES depend on the weight of the suitcase – the more the suitcase weighs, the more
25. Assuming work that there
it takes toare
liftno
thedissipative
suitcase. forces to consider, for every meter that the load is raised, two
4. Water
meters of balloons
rope must are tossed
be pulledfromup. This the roof
is dueoftoathe building,
rope passingall with over thethe bottom pulley. The work
same
done byspeed
the but
person with different
pulling must belaunch
equal angles.
to the Which
work
23. The power needed to lift the suitcase is the work required to lift the suitcase, done oneon has
the the
piano. Since the force
divided on the
by the time that
highest
piano is
it takes. speed
twice on
that impact?
exerted byIgnore
the personair resistance.
pulling, and since work is force times distance, the person
must exert their
(a) Since worksmaller
does forcedepend
over twice thethe distance that thewilllarger pulley force
on themoves the piano.
Since each balloon hasNOTthe same on kinetic
initial path, the power
energy, and each not depend path either,
balloon assuming
undergoes the the
timesameis theoverall
same change
for all paths.
in gravitational PE, each
(b) The
balloon power
will haveDOES the same depend
kineticon energy
the timeattaken. The more
the ground, and so timeeach taken, the lower the power needed.
Solutions
(c) has
one topower
Thethe Problems
same needed
speed atDOES impact. depend on the height of the table. A higher table requires more work
to lift the suitcase. If we assume that the time to lift the suitcase is the same in both cases, then
1. The force to liftand the higher
to the displacement
table takesare both
moredownwards,
power. so the angle between them is 0o.
(d) W The powermgd cos DOES depend 265 kgon 9.80 the weight
m s 2 of2.80 the m suitcase.
cos 0 o A 7.27 heavier10suitcase
3
J requires more force
G
to lift, and so requires more work. Thus the heavier the suitcase, the more power is needed to
lift it (in the same amount of time).

24. The climber does the same amount of work whether climbing straight up or via a zig-zag path,
© 2005ignoring dissipative
Pearson Education, forces.
Inc., Upper SaddleBut ifNJ.
River, a longer
All rightszig-zag
reserved.path is taken,
This material it takesunder
is protected more time tolaws
all copyright do as
the work,
they
currently
andexist.
so No
theportion
power of this material
output may be reproduced,
needed from the in any form is
climber or by any means,
less. That without
will permission
make the in writing
climb from the It is easier
easier.
publisher.
for the human body to generate a small amount137of power for long periods of time rather than to
generate a large power for a small period of time.

25. Assuming that there are no dissipative forces to consider, for every meter that the load is raised, two
meters of rope must be pulled up. This is due to the rope passing over the bottom pulley. The work
done by the person pulling must be equal to the work done on the piano. Since the force on the
piano is twice that exerted by the person pulling, 43
and since work is force times distance, the person
must exert their smaller force over twice the distance that the larger pulley force moves the piano.
ignoring dissipative forces. But if a longer zig-zag path is taken, it takes more time to do the work,
and so the power output needed from the climber is less. That will make the climb easier. It is easier
for the human body to generate a small amount of power for long periods of time rather than to
generate a large power for a small period of time.

Topic 2.3b Work done by a constant force


25. Assuming that there are no dissipative forces to consider, for every meter that the load is raised, two
meters of rope must be pulled up. This is due to the rope passing over the bottom pulley. The work
done by the person pulling must be equal to the work done on the piano. Since the force on the
piano is twice that exerted by the person pulling, and since work is force times distance, the person
must exert their smaller force over twice the distance that the larger pulley force moves the piano.
Calculation-based
Chapter 6 Questions Work and Energy

1. How much
Solutions work is done by the gravitational force when a 256kg pile driver falls 2.80m?
to Problems
2. The minimum force required to lift the firefighter is equal to his weight. The force and the
[1 mark]
displacement are both upwards, so the angle between them is 0o.
1. The force and the displacement are both downwards, so the angle
2
between them
o
is 0o.
Chapter 6 W climb
Fclimb d cos mgd cos 65.0
2 kg 9.80 m so 20.0m cos 3 0 10 4and
1.27Work J Energy
WG mgd cos 265 kg 9.80 m s 2.80 m cos 0 7.27 10 J

3. (a) See the free-body diagram for the crate as it is being pulled. Since the x
2. AThe
2. 1300-N
minimum
crate is crate rests
notforce on thehorizontally,
required
accelerating tofloor. How
lift the much
FP Ffrwork
firefighter is N
is equal
230 required
to.his The to
weight.
work move it
The force
done to at a constant
and the speed…
displacement
a.move 4.0m are
along boththe upwards,
floor so the angle
against a between
friction forcethem
of is 0o. and
230N,
Ffr FP
it across the floor is the work done by the pulling force. The angle
© 2005 Pearson Education, Inc., Upper Saddle River, NJ. All rights reserved. This material 2 ois protected under all
o copyright laws 4as they
b.W
currently exist. No4.0m
betweenportion
climb vertically?
Fthe d cos
pulling
of this
climb
mgd
force
material may cos
and
be 65.0
in any kg
the direction
reproduced, 9.80
oformotion
form mmeans,
by any iss 0 .without
20.0mpermission
cos 0 in writing
1.27from 10theJ
publisher. o 2 FN mg [2 mark]
WP FP d cos 0 230 N 4.0 m 137 1 9.2 10 J
3. (a) See the free-body diagram for the crate as it is being pulled. Since the x
crate is not accelerating horizontally, F F 230 N . The
(b) See the free-body diagram for the crate Pas it fris being lifted. Since the crate is notF work done to
accelerating
move it across vertically,
the floorthe pulling
is the workforce
doneisbythe thesame magnitude
pulling force. Theas the weight. Thefr
angle y FPFP
angle between
between the pullingthe pulling
force andforcetheand the direction
direction of motion is 0o. is 0o.
of motion
o 3 FN mg
WPP FP d cos 0 o mgd 230 N 1300 4.0Nm 4.0 1 m 9.2 5.2 10 2 J10 J
mg
(b) See the free-body diagram for the crate as it is being lifted. Since the crate is not
4. Drawaccelerating
a free-bodyvertically, thethe
diagram for pulling
crate force
as it isisbeing
the same magnitude
pushed as the
across the weight. The
floor. y FP
o x
angle between the pulling force and the direction of motion is 0
Since it is not accelerating vertically, FN mg . Since it is not accelerating . FP
o 3
WP FPFP dFcos
horizontally, fr
0 k Fmgd
N k
1300
mg N work
. The 4.0 mdone 5.2 10 Jit across the F
to move fr
mg
floor is the work done by the pushing force. The angle between the pushing mg
force and the direction of motion is 0 . o FN

4. DrawWa free-body diagram


Fpush d cos 0 o for the
mgd 1crate as it is being pushed across
0.50 160 kg 9.80 m s 10.3 m 2 the floor.
x Find the net
3. ASince
box itof
pushmass 5.0kg is accelerated
is not accelerating
k
vertically, F from rest. across
mg Since itaisfloor at a rate of 2.0m/s2Ffor 7.0s.
not accelerating
N P
work done on the box.
3
horizontally, 8.1
F 10F J F mg . The work done to move it across the F
P fr k N k fr
[2 marks]
floor is the work done by the pushing force. The angle between the pushing mg
5. Sinceand
force the the
acceleration
direction ofof the box is
motion is constant,
0o . use Eq. 2-11b to find the distance moved. FAssume
N that
the box starts from rest.o
Wpush Fpush d cos 0 1 2 k mgd 11 0.50 160 kg2 9.80 m s 2 10.3 m
x x x0 v0t 2 at 0 2 2.0 m s 2 7 s 49 m
Then the work 103 Jin moving the crate is
8.1done
W F x cos 0 o ma x 5 kg 2.0 m s 2 49 m 4.9 10 2 J
5. Since the acceleration of the box is constant, use Eq. 2-11b to find the distance moved. Assume that
the box starts from rest.
6. The first
x book
x xis already in 2position,
v t 12 at 0 1 2.0 so no
m work
s 2 7iss required
2
49 mto position it. The second book must be
moved upwards0 by 0a distance d, by2 a force equal to its weight, mg. The force and the displacement
Then
are inthe
thework
samedone in moving
direction, so thethe crate
work is
is mgd. The third book will need to be moved a distance of 2d
by the same size
W F x cos 0 force,
o so the
ma x work5 kg 2.0 m s 2 continues
is 2mgd, This 49 m through
4.9 10 2 Jall seven books, with each
needing to be raised by an additional amount of d by a force of mg. The total work done is
W mgd 2 mgd 3mgd 4 mgd 5mgd 6 mgd 7 mgd
6. The first book is already in position, so no work is required to position it. The second book must be
28mgdby 28
moved upwards 1.7 kg d,9.8
a distance bym s 2 0.043
a force equalmto its 2.0 101 Jmg.
weight, . The force and the displacement
are in the same direction, so the work is mgd. The third book will need to be moved a distance of 2d
by the same size force, so the work is 2mgd, This continues through all seven books, with each
needing to be raised by an additional amount of d by a force of mg. The total work done is
W mgd 2 mgd 3mgd 4 mgd 5mgd 6 mgd 7 mgd
28mgd 28 1.7 kg 9.8 m s 2 0.043 m 2.0 101 J .

44
© 2005 Pearson Education, Inc., Upper Saddle River, NJ. All rights reserved. This material is protected under all copyright laws as they
currently exist. No portion of this material may be reproduced, in any form or by any means, without permission in writing from the
publisher.
4. A 330-kg piano slides 3.6m down a 28° incline and is kept
from accelerating by a man who is pushing back on it
parallel to the incline. The effective coefficient of friction
Giancoli Physics: Principles with Applications, 6th Edition
μ is 0.40 and the force due to friction can be calculated
by F = μN where is the normal force.
7. Calculate:
Consider the diagram shown. If we assume that the man pushes
a. the
straight force
down exerted
on the end ofby
thethe man.
lever, then the work done by the FI
manb.(thethe work work)
“input” done by the man
is given by WonI the
F h
I I piano.
. The object moves a
c. the work done by the friction force.
shorter distance, as seen from the diagram, and so WO FO hO . lI
d. the
Equate work
the two done by
amounts of the
work.force of gravity. hI
e. the net work done FO on hI the piano.
FO hO FI hI [10 marks] lO hO
F h I O

h l F lI F
(Hint:
But byDraw the
similar free-body
triangles, diagram.
we see that I You Iwill need
, and so toO resolve
. forces parallel and perpendicular to the
O

hO lO incline.)
FI lO

8. The piano is moving with a constant velocity down the plane. FP is the FN
force of the man pushing on the piano. Ffr
y
(a) Write Newton’s 2nd law on each direction for the piano, with an FP
acceleration of 0.
x
Fy FN mg cos 0 FN mg cos mg
Fx mg sin FP Ffr 0
FP mg sin Ffr mg sin k
cos

330 kg 9.80 m s 2 sin 28o 0.40 cos 28o 3.8 10 2 N


(b) The work done by the man is the work done by FP . The angle between FP and the direction of
motion is 180o.
WP FP d cos180o 380 N 3.6 m 1.4 103 J .
(c) The angle between Ffr and the direction of motion is 180o.
Wfr Ffr d cos180o k
mgd cos 0.40 330 kg 9.8 m s 2 3.6 m cos 28o
.
4.1 103 J
(d) The angle between the force of gravity and the direction of motion is 62o. So the work done by
gravity is
WG FG d cos 62 o mgd cos 62 o 330 kg 9.8 m s 2 3.6 m cos 62 o 5.5 103 J .
(e) Since the piano is unaccelerated, the net force on the piano is 0, and so the net work done on the
piano is also 0. This can also be seen by adding the three work amounts calculated.
WNet WP Wfr WG 1400 J 4100 J 5500 J 0J

9. (a) Write Newton’s 2nd law for the vertical direction, with up as positive.
Fy FL Mg Ma M 0.10 g FL 1.10 Mg FL
(b) The lifting force and the displacement are in the same direction, so the work
done by the lifting force in lifting the helicopter a vertical distance h is
Mg
WL FL h cos 0o 1.10 Mgh .

© 2005 Pearson Education, Inc., Upper Saddle River, NJ. All rights reserved. This material is protected under all copyright laws as they
currently exist. No portion of this material may be reproduced, in any form or by any means, without permission in writing from the
publisher.
45
139
14. See the graph of force vs. distance. The work done is 25
14. See the graph
the area underofthe
force vs. distance.
graph. The work
It can be found fromdone
the is 25
20
the area under the graph.
formula for a trapezoid. It can be found from the 20
15

(N)(N)
formula for1 a trapezoid.
W 2 13.0 m 5.0 m 24.0 N 216 J 15

Fx Fx
10
W 12 13.0 m 5.0 m 24.0 N 216 J
Topic 2.3c Mechanical Energy
10
5
Giancoli 5 Principles with Applications, 6th
Physics:
0 Principles with Applications, 6 Edition
th
Giancoli Physics: Edition
0 0 2 4 6 8 10 12 14
0 2 4 6 x (m)8 10 12 14
x (m)
WG 80.0 J
15. Findd
W
Calculation-based Questions
dthe velocity from
mg
G
80.0 J 4.41
the kinetic energy, using
1.85 kg 9.80 m s
m
4.41Eq.
m 6-3. 2
15. Find the velocity
mg from 1.85thekgkinetic m s 2 using Eq. 6-3.
9.80energy,
1 2 2 KE 2 6.21 10 2121 J
1. If the KE
KE of12an mvarrow isvdoubled, 2 KE by what factor10
2 6.21 has 484 m s
26 its speed increased?
J If its speed is doubled, by
22nd
2
25. (a)
25. (a) KE the
From
From the mvfree-body
free-body v diagram
diagram mfor
for the
the load
load 5.31
being
being 10lifted,
lifted, 484Newton’s
write
write m s
Newton’s nd law for
law for
whatthe
factor 2
does its KE increase? m 5.31 10 26
F
the vertical
vertical direction,
direction, with with up up being
being positive.
positive. FTT
F F mg
2
ma 0.160 mg
[2 marks]
16. (a) Since KE F F 1
2T
1T
mv2mg , thenmav 0.160 2 KE mg m and so v KE . Thus if the kinetic energy is
16. (a) Since KE 2 mv , then v 2 KE m and so v KE . Thus if the kinetic energy is
2 3
F
doubled,
FTT the 1.16 mg
1.16speed
mg will 1.16
1.16 be285285 kg
multiplied 9.80 m s 2
kg 9.80byma sfactor3.24 3.24
of 10 10
2 3.NN
doubled, the speed will be multiplied by a factor of 2 . mg
(b)
(b) The
(b) The net
netKE
Since work
work 12donemv2 ,on
done 2
on
then the
the KEload
load is isv2found
2
found
. Thus from
from the net
thespeed
if the is doubled, the kinetic energy will be mg
net force.
force.
1
(b) SinceWKE F2 mv dd cos, then KE
o v . Thus if the speedkg is doubled, the kineticmenergy will be
2
Wnet
multiplied
net
Fnet
by cos 00o
neta factor of 4 .
0.160
0.160mg mg dd 0.160 0.160 285 285 kg 9.80 9.80 m
m ss 2 22.022.0 m
multiplied by a factor33 of 4 .
9.83
9.83 10 10 JJ
17. (c)
The work
The done on the electron is on
equal to theischange in its kinetic energy.
17. The The work
(c) work done done
work done
on the by the
byelectron
the cable
cableis on
equalthe
the toload
load
the is
change in its kinetic6 energy. 2
2. How muchW W workKE Fmust 1
mv222 be
2 d cos 0 2o done
1
o mv
2
1 1.160to
0 stop
mg
1
a
9.11
d 1250-kg
10 3131 kg
1.16 285 car travelling
1.90
kg 106 m
9.80 m at
ss222 105km/h?
22.0 1.64 18
107.13 J 4
1.64 10 J 10
m 10 4 JJ
2
W W KE F
1 T d cos 0
mv 1
mv 2
1.160
0 mg
1 d 1.16 285
9.11 10 kg 1.90 10 m s kg 9.80 m s 22.0 m 7.13
18
[2 marks]
cable
cable 2T 2 2 1 2
(d) The
The work done by gravity on
on the load is
18. (d)
The workwork
donedone
on the bycar
gravity
is equal the
to theload is
change in its kinetic energy, and so
o 2 4
18. The work done
W on
mgd the
WGG mgd cos180 car
cos180is equal
o to
mgd
mgd the change
285 in its9.80
285 kg
kg kinetic
9.80 m ssenergy,
m 2
22.0and
22.0 m so2 6.14
m 6.14 1010 4 JJ
1m s
(e) W the 1
mv22 12 theory
KEwork-energy mv12 0to find 1
1250 kg speed,
105 km h an 1m 2
5.32 105 J
(e) Use
Use the work-energy
1
2 2 1 theory
2
W KE 2 mv2 2 mv1 1 0 22 2 1 1250 to find
1
2 the
the final
final speed, with
with an
105 km h 3.6 km h
initial
s speed
initial speed of
of 0.
0.
5.32 105 J
2 kg
W
W net
KE KE
KE KE 2 mv 2 mv
2 1
1 mv
2
1 mv 2
1 3.6 km h
net 2 1 2 2 2 1

19. The force exerted by the bow on 22 the arrow


9.83 10 3
3Jis in the same direction as the displacement of the arrow.
3. A spring
19. The forcehas a stiffness
exerted
v
22W
W
bynet
o
constant
2 of,
9.83 k,
10of J440N/m.
netthevbow on the arrow is in the
2 same
00 85.8
How
8.31m
much must
direction
ss thatas
this spring beofstretched
the displacement arrow. to store
thethe
Thus W v2Fd cos 0
25J of potential energy? v
Fd
1 110 N 0.78 m 8.31m
J . But work changes the KE of
2 m 1 285 kg
arrow,Wby the
Thus cos 0mo Fd theorem.
Fd work-energy 110 N 285 0.78
kg
Thus m 85.8 J . But that work changes the KE of the
[1 mark]
arrow, by the work-energy theorem. Thus
26. The 1 2 2Fd 2 85.8 Jor compressing of the
26. The elastic
Fd PE
elastic W is
PE isKEgiven
given 2
by
by 1PE
KE mv22 212 12kx
PE12elastic
elastic
kxmv 2 where
1
where xxvis
2
is
2
the
the2distance
distance
Fd v12 of
of stretching
stretching
2 85.8 J or compressing
0 44 m s of the
spring Fdfrom W its KE
natural KE
length. 1
mv 2 1
mv 2
v m v 2
0.088 kg 0 44 m s
spring from its natural 2 length.
1 2 2 2 1 2
m
1
0.088 kg
22 PE
PEelastic 22 2525 JJ
xx
20. The work done by the ball on the glove
elastic 0.34
0.34will m
m be the opposite of the work done by the glove on the
20. The
ball.work
The donework k
k bydone theon 440
ball
theonN
440 Nball m
the
misglove equalwill be the
to the opposite
change in theofkinetic
the work doneofbythetheball.
energy glove on the
ball. WThe work KE done on the ball is
1 equal 2 1to the
KE1 ball 2 mv2 2 mv1 0 2 0.140 kg 32 m s 2 2 change 1 in the kinetic energy 2 of the ball.
72 J
on ball 2
27. Subtract the initial gravitational PE from
2 1 the 2final gravitational PE.
4. By how
Wonmuch does the gravitational potential energy of a 64kg pole-vaulter J change if his centre of
1 1
27. Subtract the
ball
KE
initial 2
KE
gravitational
1
PE mv from
2
mv
the 1
final 0 0.140
gravitational kg PE.32 m s 72
So WonPE 72
mgy J . But mgyWonballglove
mg
2
Fyon glovey2d cos 7.0 0o , because
kg
2
9.8 the
m force
s 2
1.2onmthe glove
82 J is in the same
mass
So Wonrises
PEGG about
glove
mgy
72 J . 224.0m
But Wduring
mgy 1
1
mg the jump?
Fyon22 gloveyd11 cos 07.0 o kg 9.8the
, because
2
m force
s 1.2 on m 82 Jis in the same
the glove
direction
gloveas the motion ofonthe gloveglove.
[1 mark]
direction as the motion of the glove. 72 J gravitational2 PE.
28. Subtract
28. Subtract the
72 the initial
J initial gravitational
Fon glove gravitational
0.25 m PE
PEFfrom from the final 2.9 10 N PE..
on glove the final
72 Jmgravitational
72PEJ Fon mgy 0.25 m F 0.25 2.9 10 2
N2 . 3
PEgrav
grav
mgy22 mgy
glove
mgy11 mg mg yy22 yy11 0.2564
on glove
64mkg kg 9.8 9.8 m
m ss 4.0
2
4.0 m m 2.5
2.5 10 103 JJ

29. Assume
29. that all
all of
of the
the kinetic
kinetic energy
energy of the
the carcar becomes
becomes PE of
of the
the compressed
compressed spring.
5. AAssumesled isthat initially given a shove up aoffrictionless 28.0°PE incline. It reaches
2
2
aspring.
maximum vertical height
1m ss
ofPearson
© 2005 1.35m higherInc.,
Education, than
Upperwhere it2 started.
Saddle River, 1200
NJ. All
1200 in What
kg
rights 65was
km
reserved.
kgany form
65 or
km the
hh
This initial
1m
material is speed?
protected under all copyright laws as they
©currently exist. Education,
2005 Pearson No portion Inc.,
2
of this
2
material
Upper may
Saddlemv be2 reproduced,
River,
mv NJ. All rights reserved. by any
This 3.6
means,
material km
is
3.6 km h h
without
protectedpermission
under allin writing from
copyright
4 laws the
as they [2 marks]
publisher.exist.11 mv 2 1
1 kx 2 k 8.1 10 4 N m
currently 2 mvNo 2 kx
portion of this k
material may be
2 reproduced, in any form or by any 2means, without 8.1
permission in 10
writingN m
from the
publisher. 2 2 x
x 2 141 2.2
2.2 m m
2

141
30. (a)
30. (a) Relative
Relative to
to the
the ground,
ground, the
the PEPE is
is given
given by
by
2
PE mg
PEGG mg ybooky y
yground 2.10
2.10 kg
kg 9.80
9.80 m
m ss 2 2.20
2.20 m
m 45.3
45.3 JJ
book ground

© 2005 Pearson Education, Inc., Upper Saddle River, NJ. All rights reserved. This material is protected under all copyright laws as they
© 2005 Pearson Education, Inc., Upper Saddle River, NJ. All rights reserved. This material is protected under all copyright laws as they
currently exist. No portion of this material may be reproduced, in any form or by any means, without permission in writing from the
currently exist. No portion of this material may be reproduced, in any form or by any means, without permission in writing from the
publisher.
publisher.
143
143

46
v2 2 gy1 2 9.80 m s 2 185 m 60.2 m s 135 mi h

35. The forces on the sled are gravity and the normal force. The normal force is
perpendicular to the direction of motion, and so does no work. Thus the sled’s FN
mechanical energy is conserved. Subscript 1 represents the sled at the bottom
of the hill, and subscript 2 represents the sled at the top of the hill. The ground
Chapter 6 zero location for PE y 0 . We have y 0 , v
is the 0 , and y2 1.35 m . mg and Energy
Work
1 2

Solve for v1, the speed at the bottom.


Giancoli 2 2 2
Physics: Principles with Applications, 6th Edition
1 1 1
2
mv1W mgyE1 2 Emv2 mgy F 2d cos1802
mv
o
1 1 mv
0 2 0 mgy
mgy2 1 mv 2 mgy
NC 2 3 f 3 2 2 2 2 3 3

(b) vThe1
2 gy2done 2by9.80
work m s 2 on1.35
the2 force m v 2 5.14
the pedals in one m revolution
s20.69 m sis2 equal to the tangential force times
thethat mgd 3 2 mv
circumference
1
of 2the
2
0circulard 3path of the pedals. That work is also242.7 equalmto the 10 2 m
2.4energy
Notice the k angle is not used in the calculation.
2 g k 2 9.80 m s 0.090
change of the bicycle during that revolution. Note that a vertical rise on the incline is related to
36. We assume
the distance along the incline by rise distance sin . so that the mechanical energy of the
that all the forces on the jumper are conservative,
50. (a) Apply
jumper energy conservation
is conserved. Subscript 1with no non-conservative
represents the jumper at the work. Subscript
bottom of the 1jump,
represents the ball as
and subscript 2 it
is dropped, W F 2 r PE
pedal andtansubscript 2 represents
mg y
the ball mgd
1 rev as it reaches
sin the ground. The ground is the zero
represents the jumper at the top of1 revthe jump. Call
force the ground 1 rev
the zero location for PE y 0 . We
location for gravitational PE. We have v1 0 , y1 13.0 m , and y2 0 . Solve for v2 .
6. Two y1 0 , vcars,
have railroad 2 mgd
0.70
each msin s , and
1 of mass
2
y27650kg
75 kg2.10 mand
1 9.8
. Solve
2m straveling
2 formv195km/h
5.1 ,sin
the7.8
speed
o at the bottom.
1in opposite directions, collide head-
E
2 F1tan
E 1 rev mv mgy mv mgy mgy mv22 10 2 N
2 4.5
on and 1
2
mv come
1
mgy to 2
1
rest.
1
mvHow
22 r 2
22 1
much
mgy 2
thermal
1 21
2
mv
22
energy
0
2 1 0.18 2m 2
21
is
mv produced
2
mgy
1
2
in this collision?
v2 2 2 gy1 2 9.80 m 2s 2 13.0 m 2 16.0 m s [2 marks]
v1 v2 2 gy2 0.70 m s 2 9.80 m s 2.10 m 6.45 m s
47. (b)
Use Apply
conservation energyofconservation,
energy, where butallwith non-conservative
of the kinetic energy work due to friction
is transformed included.
to thermal energy.The work
o 2
done by friction will be given by WNC Ffr d cos180 , since the force 0.238 ofmfriction
spersonis–intrampoline
the
37. (a) Since
E thereE are no dissipative
1
mv 2
E forces 1present,
2 7650 thekg mechanical
95 km energy
h of the 5.3 106 J –
opposite
initial direction
final as 2 the motion. thermal The 2 distance d over which the frictional force acts will be the
Earth combination will be conserved. The level of the unstretched trampoline is the zero level 1km h
13.0both
for m distance
the elastic of fall. With the same
and gravitational PE.parameters
Call up the as positive
above, and v2 8.00
direction. m s , solve
Subscript for the
1 represents
48. Applythe
forcethejumper
of at the topofofenergy
friction.
conservation the jump,to theand subscript
child, 2 represents
considering work done the jumper uponand
by gravity arriving
work at the
changed
7. You drop
trampoline.
into thermal a ball
W energy.
from
There a isheight
no
E ESubscript 1F represents
of
elastic d
2.0m,
PE and
1 involved
2
mv the
it
mgy
bounces
in this1 part
child atmv 2 back
the top of the
mgy
to a height
problem.
of the slide,Fand of
We 1.5m.
have
mgy 1 2mv 2 m s ,
v
d subscript 1 5.0

a. What
represents themfraction
nc
child
1 2
of bottom
its initial fr
energy
2 1
is The
lost ground
1
during isthe
2 2
thebounce?
2 fr 1
for PE y 0 . We
2 2

y1 3.0 , andatythe 02 . Solve of for


thevslide.
2, the speed upon arriving zero
at location
2 the trampoline.
b. What is the yball’s 1
2
v2speed just as it leaves the 2 ground
8.00 m after
s the bounce?
have v1 E F0fr, yE m 3.5g m 1, mv v 2.20.145
2
m1 s ,1kg mv2 9.80
and 2
y2 mgy m0 2. sSolve 1formvthe2 1.06 mvN
work1 changed
1 2
into
0 thermal
c. Where 1 1 did
2 the 2d2 1 mgy
d 2energy go? 2 2 2 13.0 1 m
mgy 2 2
energy. 2 [5 marks]
v2 v12 1 2 gy 2 1 5.01 m 2s 2 9.8 m s 2 3.0 m 9.154 m s 9.2 m s
E
51. (a) Calculate
1
E 2 the energy 2
mv mgy mv
1 of the 1ball2 at the 2
mgy
two maximum
2
W thermal heights, and subtract to find the amount of
The
energyspeed is the absolute value of v .
Wthermal “lost”.
mgy1 The 1 energy at the kgtwo9.8 heights
m s 2 is3.5 all m gravitational PE, since
2.2 m thes ball6.9
has no
102KE
J at
2 2
mv22 21.7 1
21.7 kg
(b) Now
those let subscriptheights.
maximum 2
3 represent the jumper at the maximum stretch of the trampoline. We have
2

Elost mEinitial
v2 9.154 s , y2 Efinal 0 , x2mgy0initial
, v3 mgy 0 , andfinal
x3 y3 . There is no elastic energy at position 2,
49. (a) See the free-body diagram for the ski. Write Newton’s 2nd law for
but there is elastic energy at position yinitial3. Also,
yfinal the2.0 gravitational m PEisat position 3 is negative, and
forcesEperpendicular
lost
mgyinitial to mgy
the direction
final of motion, m that
noting 1.5 there
0.25 25%
d
no yacceleration
so 3E
0 . A quadratic mgy relationship
perpendicular to the results
plane.
y from the2.0 conservation
m of energyFfr condition.
initial initial initial FN th
Giancoli E2energy 1 2
FE3 Fconservation,
mg mvcos mgy F1
kx 2 1
mgballmv 2 mgy Physics:
1 Principles with is
kx32 the ground Applications,
equal to its6final
Edition
(b) Due to N 2 2 the 2KE2N 2the
of 2cos 3just as it
3 leaves
2
PE. mg
Fmv22 k0FN 0 0k mgmgy
1
2 fr
cos3 12 ky32 2 2
1
ky32 mgy3 12 mv22 0
1
PEfinal KEground mgyfinal 2 mvground
Now use conservation 2 of
2 energy, 1
including
1 2 2
the non-conservative friction force.
m v12 Subscript
v22 2 gy11
W mg
E Eat m g 4 Fd k 1 mv
mv mgy mg 1 2 2 2
m g kmv
mv22 represents
mgy2
2
Ffski at the bottom of the
represents
vy3 NC the1 ski 2 the top offr the slope,
2gyfinal 2 9.8 m 2s 2 11.5and
2 2 2
m subscript
1
5.4
2
m s
2 the

slope.ground
The location of the 2d
2 12ski
k at the bottom of the inclinek is the zero location for gravitational
2 2
(c) The
PE energy
y 0 “lost”
.980 was 138.9
Wekghave changed
v1 0m, sprimarily
y1 d 55.56sin into heat
s y2energy
,mand 2 029.80–mthes temperature
. Write
2
the3500 m of theofball
conservation and the
energy
2 2
ground would65 have increased
kg 9.8 m s 2
slightly after the
65 kg 9.8 m s bounce. 2
Some of the
4
energy may
6.2 10 N m 65 kg 9.154 m s have been
condition, and solve for the final
changed into acoustic energy (sound waves). speed. 3500
Note m
that
Some may Ffr
F
have
k N been
mg cos
k lost due to non-elastic
2
deformation of the ball or ground.
1 2 1 2 sin10 6.2
o 104 N m
WNC KE PE 2 mv2 2 mv1 mgy2 mgy1 WNC E1 E2
© 2005 Pearson Education, Inc., Upper Saddle River,
3 NJ. All rights reserved. This material is protected under all copyright laws as they
52. Since
currently exist. the crate
No portion
2062
Ffr d cos180
moves
of
N
o 1 2 10
along
this material mv
2 may
2
the N 1 there
mgy
1be floor,
reproduced,
1
mv 2
is2 no
2in any
mgy
formchange
2 anyin
or by mgdpermission
gravitational
means, without cos mgd sinfrom the
in writing
1
mv22
8. How long will it take a 1750-W motor to lift a 315kg piano to a sixth-story window 16.0m F Pabove?
k 2
publisher. F fr
PE, so use the work-energy theorem: Wnet KE2 KE1 . There are two oforces o
v
58. The work necessary 2
2 gd sin
to lift the piano
k
cos 2 9.80
145 m s 75 m sin 22 0.090
is the work done by an upward force, equal in magnitude to the cos 22
doing work: FP , the pulling force, and Fofr F
k N k
mg , the frictional
weight of the piano. 20.69 m s W21mFd
Thus s cos 0 mgh . The average power output required to FN
lift the
mg
force.
piano isKE the 0 since the crate starts from rest. Note that the two forces doing
1 work done divided by the time to lift the piano.

work
(b) Now,do work overlevel
on the different distances. 315
mg kg 9.80 2
theremissno change
16.0 m in PE. Let us again
W mgh o ground,mgh Ff k
, and
WPP FP d P cos 0 t Wfr Ffr d fr cos180 o
mgd 28.2 s
use conservation
t t of energy,Pincluding the non-conservative1750k W fr friction force, to relate position 2
with position 3. Subscript 3 represents the ski at the end of the travel on the level, having
59. The
© 2005 Pearson18 hp is the
traveled power
a Inc.,
Education, Upper generated
distance Saddle theby
d 3 onRiver, the
level.
NJ. engine in creating
Wereserved.
All rights have a force
v2This 20.69
materialm s ,on
y2 the
is protected 0ground
, all
under 0to, and
v3 copyrightpropel
lawsyas
3
the0 car
they .
currently exist. No portion of this material may be reproduced, in any form or by any means, without permissionW in writing
Fd from the d
forward. The relationship between the power and the force is given by P
publisher. F Fv .
© 2005 Pearson Education, Inc., Upper Saddle River, NJ. All rights reserved.150 This material is protected under allt copyright t laws astthey
Thus the force to propel the car forward is found 47
currently exist. No portion of this material may be reproduced, in any form or by any means, without permission in writing from the
publisher. by F P v . If the car has a constant velocity, then
149
58. The work necessary to lift the piano is the work done by an upward force, equal in magnitude to the
weight of the piano. Thus W Fd cos 0o mgh . The average power output required to lift the
piano is the work done divided by the time to lift the piano.
9. A car generates
W mgh 18hp (1hp = mgh
746W) when
315 kgtravelling
9.80 m sat2
a16.0
steady
m 88km/h, what must the average
P
force exerted t 28.2 s
t ontthe car due to P friction and air resistance?
1750 W
[2 marks]
59. The 18 hp is the power generated by the engine in creating a force on the ground to propel the car
W Fd d
forward. The relationship between the power and the force is given by P F Fv .
t t t
Thus the force to propel the car forward is found by F P v . If the car has a constant velocity, then
the total resistive force must be of the same magnitude as the engine force, so that the net force is
zero. Thus the total resistive force is also found by F P v .
P 18 hp 746 W 1 hp
F 5.5 102 N
v 1m s
88 km h
3.6 km h

60. The power is given by Eq. 6-16. The energy transformed is the change in kinetic energy of the car.
2
1m s
1400 kg 95 km h
energy transformed KE
1
2
m v22 v12 3.6 km h
P
time t t 2 7.4 s

6.6 104 W 88 hp

550 ft lb s 4.45 N 1m
61. (a) 1 hp 1 hp 746 N m s 746 W
1 hp 1 lb 3.28 ft

1 hp
(b) 75 W 75 W 0.10 hp
746 W

1000 W 3600 s 1 J/s


62. (a) 1 kW h 1 kW h 3.6 106 J
1 kW 1h 1W
1 kW 30 d 24 h
(b) 520 W 1 month 520 W 1 month 374 kW h
1000 W 1 month 1d

© 2005 Pearson Education, Inc., Upper Saddle River, NJ. All rights reserved. This material is protected under all copyright laws as they
currently exist. No portion of this material may be reproduced, in any form or by any means, without permission in writing from the
publisher.
153

48
rushes out the open end of the balloon. That escaping gas and the balloon form a closed system, and
1. For
so the momentum
momentum to of be theconserved,
system isthe system under
conserved. Theanalysis
balloon must be “closed”
and remaining gas– acquires
not haveaany forces
momentum
equal
on and opposite
it from outside the to thesystem.momentum A coasting of thecar escaping
has air frictiongas, andand so move in the opposite
road friction on it, fordirection
example,to

Topic 2.4a Momentum Problems


the escaping
which gas. forces and thus reduce the momentum of the car. If the ground and the air were
are “outside”
considered part of the system, and their velocities analyzed, then the momentum of the entire system
4. wouldIf the rich be conserved,
man wouldbut have notfacednecessarilyaway from the momentum
the shore and of any
thrown singlethecomponent,
bag of coinslike the car.
directly away
Conceptual Since the this
Questions
from the shore, he would have acquired a velocity towards the shore by conservation of momentum.
2. Consider ice isproblem
frictionless, as a very he would light object
slide all hitting
the way andtosticking
the shore. to a very heavy object. The large
(These questions are not in an IB style but instead
object – small object combination (Earth + jumper) would have some momentum designed to check your understanding of the concept after of
thethis topic. You should
collision,
try your best to appropriately communicate your answer using prose)
5. but When duea to rocketthe very expels large gasmass in a given of the direction,
Earth, the itvelocity puts a force of theon combination
that gas. The is so small thatof
momentum it is
thenot gas-
Chapter measurable.
rocket 7 systemThus staysthe jumperand
constant, lands soon the gas
if the Earth, and nothing
is pushed to themore left, thehappens.
rocket will beLinear pushedMomentum to the
1. When right due youtorelease
Newton’s an3inflated
rd
law. So but theuntied
rocket balloon, must carry why some does kindit offlymaterial
across the to beroom?
ejected (usually
3. When exhaustyou from release
someankind inflated
of engine)but untied to changeballoon at rest, the gas inside the balloon (at high pressure)
direction.
17. rushes
When you out the are open
lyingend flat of onthe theballoon.
floor, your That CM escaping
is insidegas of and the balloon
the volume of yourformbody. a closedWhen system,you sit and
6. so up the
The on momentum
airthebag greatly
floor with ofincreases
the system
your legsthe isamount
extended,conserved. of time
your CMThe isballoon
over whichand
outside remaining
thethe
of stopping
volumeforce gas
of youracquires
actsbody. a momentum
on the driver. If
Chapterequal
a hard 7 and object opposite
likeCM a to the momentum
steering wheel or windshield of the escaping is what gas,stops
and so themovedriver, in then
the opposite
aCM direction
largeLinear
force Momentum
is to
exerted
the
overescaping
a very short gas. time. If a soft object like an air bag stops the driver, then a much smaller force is
exerted over a much longer time. For instance, if the air bag is able to increase the time of stopping
17. If
4.
GiancolibyWhen a factor
the youman
rich are10,
of lying
wouldthenflat theonaverage
have the floor,
faced force
away your onCM
from the isshore
theperson insidewill
andof thrown
thedecreased
be
Physics: volume ofby
the bag
Principles your
of a coins
with body.
factor ofWhen
10. This
directly
Applications, 6you
th
away sit
Edition
2. Cars up
greatly onusedthe
reducesto
floorbe built
with
the youras
possibility rigid
legs as possible
extended,
of serious your
injuryto
from the shore, he would have acquired a velocity towards the shore by conservation of momentum.
withstand
CM or is
death.outside collisions.
of the Today,
volume of though,
your body. cars are designed to
have
Since“crumple zones”
the ice is frictionless,
CM
thathecollapsewould slide upon allimpact.
the way What is the advantageCM
to the shore. of this new design?
7.
18. bat.“Crumple
The But engine ifzones”
vdoes
B ,are
0not similar
pitchedto
thedirectly air bags
accelerate
ball in that
the
situation, car.wethey see increase
The engine
that the putsthe time
magnitudea force ofofinteraction
onvtheB
2vA ,during
driving soathe ball
andwheels,
5. movesWhen
makingaaway
collision, rocket
themandwith expels
rotate.
therefore gas in a given direction, it puts a force
greater speed. If, for example, the pitching speed of the ball was about Newton’s
The wheels
lower the then
average push backwards
force required on forthe on
the that
roadway
change gas.as
in The
they
momentummomentum
spin. Thethat of
thethe
twice car gas-
the
3rd lawsystem
rocket
undergoes in the
reaction stays constant,
tocollision.
this force isand thesoforward-pushing
if the gas is pushed of the to roadway
the left, the rocket
on the wheels,will be pushed
which to the
accelerates
speed at which the batterrd could swing the bat, then we would have vB 4vA . Thus the ball has
right
the car. dueSo to itNewton’s
is the (external) 3 law. road So the rocketthat
surface must carry some
accelerates thekindcar. of material to be ejected (usually
greater
exhaust speed after being struck, and thus it is easier to hit a home run. This is similar to the
8.
3. From Eq.from
Is“gravitational
it possible 7-7 forsome
for an a 1-Dkindelastic
object
of engine) collision,
toaccelerate
receive
to change vlarger
athe vdirection.vB vA . from
B impulse
Let “A” represent
aasmall force the bat,from and let “B” force?
on thethan a large
A
18. The
19. represent engine
The motion of slingshot”
does the notcenter effect
directlyof mass discussed
of the in
rocket problem
car. willThe 85.
engine
follow the puts
original forceparabolic driving
path, wheels,
both before
6. Explain.
making the ball.
them rotate. TheThe positive
wheels direction
thenpiecepush will be the (assumed
backwards on thethe horizontal)
roadway direction
asforce
they spin. that
The the bat is
Newton’s
The
and air bag
after greatly
explosion. increases
Each the amount
individual of of time theover rocket whichwill followstopping
awith
separate acts on
path afterthethe driver. If
9. The moving
rd impulsewhen the
islike
the ball is hit. We assume the batter can swing the bat equal strength in either
3hard law
aexplosion, reaction
object but toaproduct
since this
theforce
steering ofwheel
explosion isthetheforce
or
was and
forward-pushing
windshield
internal the time toisthe duration
ofsystem
what thestops that
roadwaythethe onforce
driver,
(consisting the isthe
then
of applied.
wheels,arocket),
largewhichSothethe
force accelerates
is exerted
center of
case,
impulse
the car.
over
mass so that
from
aofvery Sothe
all va is
small the
it Aisexploded
short the
time. same
force
(external) in
applied
a softroad
Ifpieces both
object
will pitching
over
surface a
like an
follow long situations.
thatair
the time can
accelerates
bag stops
original Because
be larger the bat is so much
car. then a much smaller by
path.the driver, than the impulse heavier
applied than
force the
a large
is
force
exerted
ball, we over over a small
assume a much thattime. longer
vA vAtime. – theFor speed instance,
of the bat if the air bagchange
doesn’t is ablesignificantly
to increase the timethe
during of collision.
stopping
19. by The motionofof10,
a factor thethen center the of mass of
average forcethe on rocketthe person will follow will be thedecreased
original parabolic
by a factor path, bothThis
of 10. before
10. The Then the velocity
momentum of ofanthe objectbaseballcan after
be beingof
expressed hitin vB of
isterms vAwill
its vkinetic vB energy,
2separate
vA vas If vBafter
.follows. 0 ,thethe ball
Calculation-based
and after
greatly reduces
Solutions
explosion,
tossed up
explosion.
to Problems
but
into
theQuestions
since
the 2 2the
Each individual
explosion
piece
possibility of serious injury or death.
was 2internal
vmB 1 2mv
the
vAto
rocket
2 –thethesystem
Afollow
(consisting
a
of the
B path
rocket), the center of theof
p mv m v by the
air mbatter,
mv 2 then 2
ball
2mKE moves . away with twice the speed
©
mass
7.2005“Crumple of allzones”
the Inc.,
exploded
are similar pieces
Saddleto
will
air bagsfollow
Allin
the
that original
they increasepath.the time ofunder interaction during
laws asathey
4.
1. AThus pPearsonmvEducation,
constant friction kgUpper
force sRiver,
ofsame
25N
NJ.
actskg on
rights reserved.
a 65kg
This material
orskier for
is protected
20s. What
all copyright
inismass
the skier’s change in velocity?
exist.ifNotwo 0.028
objects
of this have
8.4 m
the 0.24
kinetic m
inenergy,
sform then thefor one with more has the greater
collision,
currently and
portion therefore material lower
may thereproduced,
be average force
any required by any means,the change
without permissionmomentum
in writing that
from thethe car
momentum.
publisher. [2 marks]
undergoes in the collision.
164
Solutions
2.
11. From Newton’s
Consider
to Problems
two objects, second each law,withp theFsame t . For a constant
magnitude mass object,moving
of momentum, p minvopposite . Equatedirections.
the two
8. They From
expressionsEq. 7-7
have a for for a p.
total momentum1-D elastic collision,
of 0. If theyAcollide v v B
v
andB
v
have
A
. Let “A” represent the
a totally inelastic collision, in which bat, and let “B” they
1. stick p together,
represent mv the0.028 thenkg
ball. their8.4
The mFscommon
positive
final 0.24 speed
tdirection kgwill m must sbe thebe(assumed0 so that horizontal)
momentumdirection is conserved. that the Butbat is
since
moving
they F t
are when m
not moving v
the ballafter v
is hit.
the We .assume
collision, they thehave batternocan swingenergy,
kinetic the batand with soequal
all ofstrength in either
their kinetic energy
m
has
case, been
so lost.v A is the same in both pitching situations. Because the bat is so much heavier than the
that
2. IfFrom the skier
Newton’s movessecond to the law, right, then p Fthet speed . For awill decrease,
constant mass because
object,thepfriction m v force . Equateis tothe thetwo left.
12. Theball, we assume
expressions
turbine for tthat
F
blades . vA25 N
pshould vbeA designed
–20thes speed of
so7.7thatthe batwater
doesn’t change significantly during thethat collision.
v m thes rebounds. If the water rebounds, means
that a larger
Then the velocity momentum
m of the baseball change
65 kg for the water has occurred than if it
F t after being hit is vB vA vA vB 2vA vB . If vB 0 , the balljust came to a stop. And if there
isThe F
a larger t m
momentum v v
change .
for the water, there will also be a larger momentum change for the
tossed skier
up into loses the 7.7airmby of speed.
s the m then vB 2vA – the ball moves away with twice the speed of the
batter,
blades, making them spin faster.
© 2005 If the skier
Pearson Education, moves to theSaddle
Inc., Upper right, then
River, NJ.theAllspeed will decrease,
rights reserved. because
This material theunder
is protected friction forcelaws
all copyright is to the left.
as they
3. Choose
exist. Nothe
5. A(a)0.145kg direction
baseball from
pitched
25 N atthe batter
39.0m/s to the pitcher to be the
ismhits onisaunchanged. positive
horizontal line direction. Calculateback ataverage
the
20of s the momentum The drive straight ofthe pitcher at
currently portion of this
F t componentmaterial may be reproduced, in any form or by any means, without permission in writing from the
13.
publisher. Thev downward 7.7 horizontal component
force from the change in momentum of the ball.
52.0m/s.momentum If themcontact
changes time 65 kgbetween
from rightward to theleftward.
bat
164andThus the ball is 3.00x10-3s,
the change in momentumcalculate is tothe
the average
left in force
p F t m v
between
The skierthe
the picture.
losesbat7.7 andm sthe of ball
speed. during contact.
(b) Since thevforce on the wall52.0 m s 39.0
is opposite that m on sthe ball, the force on the wall is to the right. [2 marks]
F m 0.145 kg 3
4.40 103 N, towards the pitcher
3. Choose the direction t from the batter 3.00to the 10pitcher
s to be the positive direction. Calculate the average
14. (a) The momentum of the ball is not conserved during any part of the process, because there is an
force from the change in momentum of the ball.
external force acting on the ball at all times – the force of gravity. And there is an upward force
4. The throwing p F of t the m package
v is a momentum-conserving action, if the water resistance is ignored.
on the ball during the collision. So considering the ball as the system, there are always external
Let “A” represent the boat
forces on vit, and so its momentum and childmtogether,
52.0 sis not 39.0 andm let
s “B” represent
conserved. the package. Choose the
3
directionF m
that the 0.145
package
(b) With this tdefinition of the system, kgis thrown as the positive 4.40
direction. 10
Apply N, towards the pitcher
conservation of momentum,
withEarth-ball
the initial system velocityisofconserved
both objects 3.00all
being
of the
10 3
s forces are internal, and so the momentum of the
0.
during the entire process.
(c) The answer here is the same as formpart
p initial
p final
m A
m B
v v mv
A A (b).B B
4. The throwing of the package is a momentum-conserving action, if the water resistance is ignored.
mB v B 6.40and kg child10.0 m s
15. InLet “A”
v A torepresent
order maintainthe boat
balance, your CMtogether,must be and let “B”
0.901m
located s represent
directly abovethe package.
your feet. IfChoose
you have thea heavy
direction
load in your arms, that m the
A
package 26.0 is thrown
kg 45.0 as kgthe positive direction. Apply
your CM will be out in front of your body and not above your feet. So you lean conservation of momentum,
withboat
The
backwards the initial
and child
to get velocity
yourmoveCMofindirectly
both objects
the oppositeabove being
your0.feet.as Otherwise,
direction the thrown you package.
would fall over forwards.
p
© 2005 Pearson Education,
initial
p m
Inc., Upper Saddle
final
m
A River,
v m v
B NJ. All rights
mB49
A A reserved.
vB This material is protected under all copyright laws as they
currently exist. No portion of this material may be reproduced, in any form or by any means, without permission in writing from the
16. The 1-m length
publisher. m vof pipe is 6.40uniform
kg 10.0 – it mhass the same density throughout, and so its CM is at its
1. p mv 0.028 kg 8.4 m s 0.24 kg m s

2. From Newton’s second law, p F t . For a constant mass object, p m v . Equate the two
6. Aexpressions
child in a boat
for throws
p. a 6.40kg package out horizontally with a speed of 10.0m/s. Calculate the
velocity of the boat immediately
F t after, assuming it was initially at rest. The mass of the child is
F t m v v .
26.0kg and that of the boatmis 45.0kg. Ignore water resistance.
If the skier moves to the right, then the speed will decrease, because the friction force is to the left. [2 marks]
F t 25 N 20 s
v 7.7 m s
m 65 kg
The skier loses 7.7 m s of speed.

3. Choose the direction from the batter to the pitcher to be the positive direction. Calculate the average
force from the change in momentum of the ball.
p F t m v
Giancoli v 52.0 m s 39.0 m s Physics: th
F m 0.145 kg 3
4.40 10Principles
3 withthe
N, towards Applications,
pitcher 6 Edition
t 3.00 10 s

5.
4. TheThe force
throwing on theof gasthe can be found
package from its change in momentum.
is a momentum-conserving action, if the water resistance is ignored.
Giancoli
Let “A” p
represent v m
the boat m
and child Physics: Principles with Applications, 6th Edition
F v 4.0 10 m s 1500 kg srepresent
together,
4 and let “B” 6.0 10 the7 Npackage.
downward Choose the
direction that t the package
t ist thrown as the positive direction. Apply conservation of momentum,
withforce
The the initial
on thevelocity
rocket isoftheboth 3rd law0.pair (equal and opposite) to the force on the gas, and
objects being
Newton’s
5. The force on the gas can be found
mB v mits
from v change
mB vBin momentum.
so is p6.0 p7
initial 10 final
p Nvupward m
m.mA A A

F mB v B v kg 4.0
6.40 10.0 m 10s4 m s 1500 kg s 6.0 10 7 N downward
vA t t t 0.901m s
6. The
The tackle
force on will
mAberocket
the analyzed
26.0
is the askga one-dimensional
45.0 kg3rd law pair
Newton’s momentum
(equal andconserving
opposite) situation.
to the forceLet on“A”the gas, and
represent
The boat the
and halfback, and “B” represent the tackling
7child move in the opposite direction as the thrown package.
cornerback.
so is p6.0 10p N upward .v m v
© 2005 Pearson Education,
initial Inc., UppermSaddle
final A A River, B BNJ. Allmrights
A
mreserved.
B
v This material is protected under all copyright laws as they
currently exist. No portion of this material may be reproduced, in any form or by any means, without permission in writing from the
7. AThe
12,600-kg
publisher.
6. tackle
mA vrailroad
will
mB vB car 95
A be analyzed travels aloneson a85
kg 4.1m
as a one-dimensional level
kg frictionless
5.5 m s track with
momentum conserving a constant speed of 18.0m/s.
v 4.8 m s situation. Let “A”
Arepresent
5350-kgthe load, initially
mhalfback,
A
m B andat“B”rest, is95
dropped
represent onto
kg the
166
kg the tackling
85 car. What will be the car’s new speed?
cornerback.
pinitial pfinal m A v A mB v B mA mB v [2 marks]
7. Consider the horizontal motion of the objects. The momentum in the horizontal direction will be
m A v A mB v B 95 kg 4.1m s 85 kg 5.5 m s
conserved.
v Let “A” represent the car, and “B” represent the load. 4.8 Thempositive
s direction is the
direction of m mB
theA original motion of the 95car.
kg 85 kg
pinitial pfinal m A v A mB v B mA mB v
7. Consider the horizontal motion of the objects. The momentum in the horizontal direction will be
m A v A mB v B 12, 600 kg 18.0 m s 0
conserved.
v Let “A” represent the car, and “B” represent12.6 themload.
s The positive direction is the
direction of the m m
A original
B
12, 600 kg
motion of the car. 5350 kg
pinitial pfinal m A v A mB v B mA mB v
8. Consider the motion in one dimension, with the positive direction being the direction of motion of
8. Athe9300-kg
first mboxcar mtravelling
v A “A” v at 15.0m/s
12, 600 kg 18.0strikes
m s a0 second boxcar at rest. The two stick together and
v car.A Let B Brepresent the first car, and “B” represent the second car. Momentum will be
12.6 m s
move
conservedoff withinmthe
A
a collision.
common
mB speed
12,
Note 600 of v6.0m/s.
kg
that 5350
0 . What
kg is the mass of the second car?
[2 marks]
B

pinitial pfinal mAv A mB vB mA mB v


8. Consider the motion in one dimension, with the positive direction being the direction of motion of
the first car. ALetA “A” represent the first car, and6.0
m v v 9300 kg 15.0 m s “B”m represent
s the second car. Momentum will be
mB 1.4 104 kg
conserved in thev collision. Note that6.0 vB m 0s .
pinitial pfinal mAv A mB vB mA mB v
9. The force stopping the wind is exerted by the person, so the force on the person would be equal in
magnitude mA opposite
v A v in 9300 kg to15.0
themforce
s 6.0 m s the wind.4 Calculate the force from Eq. 7-2,
mB and direction stopping 1.4 10 kg
in magnitude only. v 6.0 m s
mwind 40 kg s 1m s
9. The force stopping2 the1.50 windmis exerted
0.50 m by the
30 kg s so vthe
person, 100 km h 27.8 m s
wind force on the person would be equal in
t m 3.6 km h
magnitude and opposite in direction to the force stopping the wind. Calculate the force from Eq. 7-2,
in magnitude only. pwind mwind vwind mwind
Fon Fon vwind 30 kg s 27.8 m s
mperson
wind
40 kg s t
wind t t 1m s
2
1.50 m 0.50 m 30 kg s vwind 100 km h 27.8 m s
t m
833 N 8 10 N 2 3.6 km h

The typical pfrictional


mwind
forcevwind
is Ffr mwinds mgv 1.0 3070 mms 2s 690 N , and so we
Fon maximum Fon wind
wind
kgkgs 9.8
27.8
person wind t t t
see that Fon Ffr – the wind is literally strong50 enough to blow a person off his feet.
person833 N 8 10 2 N
9. Suppose the force acting on a tennis ball (mass 0.060kg) points in the +x direction and is given by
Chapter 7 Linear Momentum
the graph as a function of time. Use graphical methods to estimate
a. the total impulse given the ball, and
(b) b.Thethe velocity ofis the
deceleration ballfrom
found after being struck,
Newton’s 2nd law.assuming the ball is being served so it is nearly at
rest initially. F 1.389 105 N
F ma a 93 m s 2 [2 marks]
m 1500 kg

19. Call east the positive direction.


(a) poriginal mvoriginal 95 kg 4.0 m s 3.8 102 kg m s
fullback fullback

(b) The impulse on the fullback is the change in the fullback’s momentum.

pfullback m vfinal vfinal 95 kg 0 4.0 m s 3.8 10 2 kg m s


fullback fullback

(c) The impulse on the tackler is the opposite of the impulse on the fullback, so 3.8 10 2 kg m s
(d) The average force on the tackler is the impulse on the tackler divided by the time of interaction.
p 3.8 102 kg m s
F 5.1 102 N
t 0.75 s

20. (a) The impulse given the ball is the area under the F vs. t graph. Approximate the area as a
triangle of “height” 250 N, and “width” 0.01 sec.
p 12 250 N 0.01 s 1.25N s
(b) The velocity can be found from the change in momentum. Call the positive direction the
direction of the ball’s travel after being served.
p 1.25N s
p m v m v f vi v f vi 0 21m s
m 6.0 10-2 kg

21. Find the velocity upon reaching the ground from energy conservation. Assume that all of the initial
potential energy at the maximum height hmax is converted into kinetic energy. Take down to be the
positive direction, so the velocity at the ground is positive.
1 2
mghmax 2
mvground vground 2 ghmax
When contacting the ground, the impulse on the person causes a change in momentum. That
relationship is used to find the time of the stopping interaction. The force of the ground acting on the
person is negative since it acts in the upward direction.
mvground
F t m 0 vground t
F
We assume that the stopping force is so large that we call it the total force on the person – we ignore
gravity for the stopping motion. The average acceleration of the person during stopping a F m
is used with Eq. 2-11b to find the displacement during stopping, hstop .
2
1 2
mvground 1
F mvground
y y0 v0t 2
at hstop vground 2
F m F
2 2 2
mv ground 1
mv ground 1
mvground ghmax m Fhstop
hstop 2 2
hmax
F F F F mg

© 2005 Pearson Education, Inc., Upper Saddle River, NJ. All rights reserved. This material is protected under all copyright laws as they
currently exist. No portion of this material may be reproduced, in any form or by any means, without permission in writing from the
publisher.
170

51
force is divided between both legs. Thus the critical average stopping force is twice the breaking
strength
22. Let of a single
A represent leg.
the 0.440-kg ball, and B represent the 0.220-kg ball. We have vA 3.30 m s and
6
vB 0 . Use Eq. Fhstop
7-7 to2obtain
Fbreak hstop
a relationship N m 2 the2.5
2 170 10between 10 4 m 2 0.60 m
velocities.
Topic 2.4b Momentum and Energy Problems
hmax
vA vB mg
vA vB mg vB vA vA 75 kg 9.8 m s 2
69 m

Substitute this relationship into the momentum conservation equation for the collision.
Calculation-based
22. Let AmArepresent
vA mB vBthe Questions
m v mBball,
0.440-kg
A A
vB and B mArepresent
vA mA vthe
A
m vA vball.
0.220-kg
B A We have vA 3.30 m s and
vB 0 . Usem Eq.mB7-7 to obtain
0.220akg
relationship between the velocities.
vA iceA puck,
1. A 0.450kg moving
vA east with
3.30amspeed
s of 3.00m/s,
1.10 m s easthas a head on collision with a 0.900kg
vA vm mvBA vB 0.660 vkg vA vA
puck initially at rest. Assuming a perfectly elastic collision, what will be the speed and direction of
BA B

Substitute this relationship into the momentum conservation equation for the collision.
each vobject
B
vA after
vA the
3.30collision?
m s 1.10 m s 4.40 m s east
mA vA mB vB mA vA mB vB mA vA mA vA mB vA vA
[4 marks]
23. Let A representmA them0.450-kg 0.220
puck,kg and3.30
let B
vA B
vA mrepresent
s 1.10the
m 0.900-kg
s east puck. The initial direction of
m m
puck A is the positive
A
0.660 kg
B direction. We have vA 3.00 m s and vB 0 . Use Eq. 7-7 to obtain a
relationship
vB vbetween the velocities.
vA 3.30 m s 1.10 m s 4.40 m s east
A
vA vB vA vB vB vA vA
23. Substitute this relationship
Let A represent intopuck,
the 0.450-kg the momentum conservation
and let B represent equationpuck.
the 0.900-kg for theThe
collision.
initial direction of
mA vA mB vB mA vA mB vB mA vA mA vA mB vA vA
puck A is the positive direction. We have vA 3.00 m s and vB 0 . Use Eq. 7-7 to obtain a
mAbetween
relationship mB the velocities.
0.450 kg
vA vA 3.00 m s 1.00 m s 1.00 m s west
vA vm BA
mvBA vB 1.350vkg B
vA vA
Substitute this relationship into the momentum conservation equation for the collision.
vB vA vA 3.00 m s 1.00 m s 2.00 m s east
mA vA mB vB mA vA mB vB mA vA mA vA mB vA vA
mA themball
24. Let A represent 0.450 kg m/s, and call that direction the positive direction. Let B
vA
2. Two billiard
B
vAmoving
balls of equal
at 2.00
mass undergo3.00 m s 1.00elastic
a perfectly m s head-on
1.00 m s collision.
west If one ball’s initial
represent themball mB
moving 1.350m/s
at 3.00 kg in the opposite direction. So vA 2.00 m s and
speed was 2.00m/s, and the other’s was 3.00m/s in the opposite direction, what will be their
A

vB v3.00
speeds vmA the
Bafter
s v. AUse Eq.m7-7s to1.00
collision?
3.00 obtain
m sa relationship between the velocities.
2.00 m s east
vA vB vA vB vB 5.00 m s vA [4 marks]
24. Substitute
Let A represent the ball moving
this relationship at momentum
into the 2.00 m/s, and call that direction
conservation equationtheforpositive direction.
the collision, Letthat
noting B
mB . the ball moving at 3.00 m/s in the opposite direction. So vA 2.00 m s and
mrepresent
A

vB mA3.00
vA mB vsB. Use
mA vEq.
A
mB vto
7-7 v vB vA vBbetween the velocities.
B obtain Aa relationship

v1.00
A
vm
B s vvAA vvBA 5.00 vmB s 5.00 m 2svA vA 6.00 m s vA 3.00 m s
Substitute this relationship into the momentum conservation equation for the collision, noting that
mA vB mB5.00
. m s vA 2.00 m s
The two
mAballs
vA m have
vB exchanged
mA vA mvelocities.
v vAThisvBwillvAalways
vB be true for 1-D elastic collisions of
objects of equalBmass. B B

1.00 m s vA vA 5.00 m s 2vA 6.00 m s vA 3.00 m s

vB 5.00 m s vA 2.00 m s
The two
© 2005 Pearson balls have
Education, exchanged
Inc., Upper velocities.
Saddle River, This
NJ. All rights will This
reserved. always beistrue
material for under
protected 1-D elastic collisions
all copyright of
laws as they
currentlyobjects
exist. Noof equal
portion mass.
of this material may be reproduced, in any form or by any means, without permission in writing from the
publisher.
171

© 2005 Pearson Education, Inc., Upper Saddle River, NJ. All rights reserved. This material is protected under all copyright laws as they
currently exist. No portion of this material may be reproduced, in any form or by any means, without permission in writing from the
publisher.
171

52
3. A 10,000kg railroad car, A travelling at a speed of 24.0m/s strikes an identical car, B, at rest. If the
cars lock together as a result of the collision, calculate:
a. the common speed after the collision.
b. how much of the initial kinetic energy is transformed to thermal or other forms of energy.
[4 marks]

53
Markscheme-Topic 2: Mechanics
1. C
2. A
3. B
4. B
5. A
6. A
7. B
8. C
9. C
10. C
11. A
12. C
13. B
14. D
15. A
16. A
17. A
18. D
19. A
20. B
21. C
22. D
23. C
24. D
25. B
26. B
27. D
28. B
29. C
30. D
31. B
32. C
33. C
34. A
35. B
36. B
37. C
38. A
1

54
39. B
40. D
41. C
42. D
43. A
44. C
45. B
46. D
47. A
48. A
49. B
50. A
51. A
52. B
53. D
54. C
55. C
56. C
57. D
58. A
59. B
60. C
61. B
62. C
Short answer questions
1. Linear motion
(a) change in velocity / rate of change of velocity;
per unit time / with time; (ratio idea essential to award this mark) 2

(b) (i) acceleration is constant / uniform; 1

t=
2s
and t =
(v − u ) ;
(ii)
(u + v ) a

clear working to obtain v2 = u2 + 2as; 2


(c) (i) 1.96 = 12 ? 9.81 ? t2;
t = 0.632s; 2
(ii) time to fall (1.96 + 0.12)m is 0.651s;
shutter open for 0.019s; 2
If the candidate gives a one significant digit answer treat it as an SD-1.
Award [0] if the candidate uses s = 12 at2 and s = 12cm.
[9]

55
v2
2. (a) (i) h= ;
2g
to give h = 3.2 m; 2

(ii) 0.80 s; 1
(b) time to go from top of cliff to the sea = 3.0 – 1.6 = 1.4 s;
recognize to use s = ut + 12 at2 with correct substitution,
s = 8.0 × 1.4 + 5.0 × (1.4)2;
to give s = 21 m; 3
Answers might find the speed with which the stone hits the sea
from v = u + at, (42 m s–1) and then use v2 = u2 + 2as.
[6]
–1
3. (a) vV = 8.0 sin 60 = 6.9 m s ;
v2
h= ;
2g
to give h = 2.4 m ; 3
Award [1] if v = 8.0 m s–1 to get h = 3.2 m is used.

(b) vH = 8.0 cos 60;


range = vHt = 8.0 cos 60 × 3 = 12 m; 2

Award [1] if v = 8.0 m s–1 to get R = 2.4 m is used.


[5]

4. (a) use 12 mv2 + mgh = 1


2
mv2 or some statement that conservation of
energy is used;
= 1
2
(15)2 + 700 = 1
2
V2;
to give V = 40ms−1 2

(b) appreciate that the horizontal velocity remains unchanged;

−1 15
so that θ = cos = 68° ; 2
40
Accept alternative methods of solutions for (a) and (b) based on vertical
component of velocity calculations and vector addition of components.
[4]
40
5. (a) (i) vx = = 8.0 m s–1 1
5.0
Accept use of other values leading to the same answer.

56
(ii) vy = 0 = u0y – 10 × 2.5;
u0y = 25 m s–1; 2
Accept use of other values leading to the same answer.

(iii) the x and y components of displacement at 3.0 s are 24 m, 30 m;


so the magnitude is 24 2 + 30 2 = 38 m 2

(b) maximum height reached is less;


asymmetric with shorter range; 2
[7]
6. (a) zero; 1

(b) resultant vertical force from ropes = (2.15 ? 103 − weight) = 237N;
equating their result to 2T sin50;
ie 2T sin 50 = 237
calculation to give T = 154.7N ? 150N; 3
Accept any value of tension from 130 N to 160 N. Award [2] for missing
factor of 2 but otherwise correct ie 309 N.

(c) correct substitution into F = ma;


237
to give a = =1.21 ms − 2 ; 2
1.95 ×10 2
Watch for ecf.
NB Depending on value of g answer will vary from 1.0(3) ms−2
to 1.2(3) ms−2 all of which are acceptable.
(d) statement that air friction increases with increased speed seen / implied;
in 10 seconds friction goes from 0 N to 237 N / force increases from zero
until it equals the net upward accelerating force; 2
[8]

7.

57
[14]
8.
(a) nature of the surfaces; normal reaction; relative motion of the surfaces; [2 max]
(b) friction is the frictional force between an object and a surface / two surfaces;
static friction is (the frictional force) when the object/surfaces are at rest;
dynamic friction is(the frictional force) when the object is sliding / one of the surfaces is sliding / moving
with respect to the other;
some additional comment e.g. friction varies from zero to maximum / maximum value of static friction
always greater than kinetic friction; [3 max]
Award [1 max] for an answer such as “friction force on an object at rest and friction force on a moving
object”. Some appreciation that it is friction between two surfaces is required.

[9]

9. (a) mass × velocity; 1

(b) (i) momentum before = 800 × 5 = 4 000 N s;


momentum after = 2 000v;
conservation of momentum gives v = 2.0 m s–1; 3

(ii) KE before = 400 × 25 = 10 000 J KE after = 1 000 × 4 = 4 000 J;


loss in KE = 6 000 J; 2

58
(c) transformed / changed into;
heat (internal energy) (and sound); 2
Do not accept “deformation of trucks”.
[8]
10. (a) Note: for part (i) and (ii) the answers in brackets are those arrived at if 19.3 is
used as the value for the height.
(i) height raised = 30 sin 40 = 19 m;
gain in PE = mgh = 700 × 19 = 1.3 × 104 J (1.4 × 104 J); 2

(ii) 48 × 1.3 × 104 J = 6.2 × 105 J (6.7 × 105 J); 1

(iii) the people stand still / don’t walk up the escalator


their average weight is 700 N / ignore any gain in KE of the people; 1 max
(6.2 × 10 5 )
(b) power required = = 10 kW (11kW);
60
Pout Pout
Eff = , Pin = ;
Pin Eff
Pin = 14 kW (16 kW); 3
[7]
3
11. (a) momentum of object = 2 × 10 × 6.0;
momentum after collision = 2.4 × 103 × v;
use conservation of momentum, 2 × 103 × 6.0 = 2.4 × 103 × v;
to get v = 5.0 m s–1; 4
Award [2 max] for mass after collision = 400 kg and
v = 30 m s–1.

(b) KE of object and bar + change in PE = 1.2 × 103 × 25 + 2.4 × 103 × 10 × 0.7533;
use ∆E = Fd, 4.8 × 104 = F × 0.75;
to give F = 64 kN;
Award [2 max] if PE missed F = 40 kN.
or
v2
a= ;
2s
F – mg = ma;
to give F = 64kN; 3
Award [2 max] if mg missed.
[7]

59
Topic 2 (New) [90 marks]
An elastic climbing rope is tested by fixing one end of the rope to the top of a crane. The other
end of the rope is connected to a block which is initially at position A. The block is released from
rest. The mass of the rope is negligible.

The unextended length of the rope is 60.0 m. From position A to position B, the block falls freely.

1a. At position B the rope starts to extend. Calculate the speed of the block at position B. [2 marks]

Markscheme
use of conservation of energy
OR
v2 = u2 + 2as

v = «√2 × 60.0 × 9.81» = 34.3 «ms–1»

[2 marks]

At position C the speed of the block reaches zero. The time taken for the block to fall between B
and C is 0.759 s. The mass of the block is 80.0 kg.

1b. Determine the magnitude of the average resultant force acting on the block between B [2 marks]
and C.

60
Markscheme
use of impulse Fave × Δt = Δp
OR
use of F = ma with average acceleration
OR
F = 80.0×34.3
0.759

3620«N»

Allow ECF from (a).


[2 marks]

1c. Sketch on the diagram the average resultant force acting on the block between B and [2 marks]
C. The arrow on the diagram represents the weight of the block.

Markscheme
upwards
clearly longer than weight

For second marking point allow ECF from (b)(i) providing line is upwards.
[2 marks]

1d. Calculate the magnitude of the average force exerted by the rope on the block [2 marks]
between B and C.
61
Markscheme
3620 + 80.0 × 9.81
4400 «N»

Allow ECF from (b)(i).


[2 marks]

For the rope and block, describe the energy changes that take place

1e. between A and B. [1 mark]

Markscheme
(loss in) gravitational potential energy (of block) into kinetic energy (of block)

Must see names of energy (gravitational potential energy and kinetic energy) – Allow for
reasonable variations of terminology (eg energy of motion for KE).
[1 mark]

1f. between B and C. [1 mark]

Markscheme
(loss in) gravitational potential and kinetic energy of block into elastic potential energy of
rope

See note for 1(c)(i) for naming convention.


Must see either the block or the rope (or both) mentioned in connection with the appropriate
energies.
[1 mark]

1g. The length reached by the rope at C is 77.4 m. Suggest how energy considerations [2 marks]
could be used to determine the elastic constant of the rope.

62
Markscheme
k can be determined using EPE = 12 kx2

correct statement or equation showing


GPE at A = EPE at C
OR
(GPE + KE) at B = EPE at C

Candidate must clearly indicate the energy associated with either position A or B for MP2.
[2 marks]

A small ball of mass m is moving in a horizontal circle on the inside surface of a frictionless
hemispherical bowl.

The normal reaction force N makes an angle θ to the horizontal.

2a. State the direction of the resultant force on the ball. [1 mark]

Markscheme
towards the centre «of the circle» / horizontally to the right

Do not accept towards the centre of the bowl


[1 mark]

63 length to represent the weight of the


On the diagram, construct an arrow of the correct
2b. On the diagram, construct an arrow of the correct length to represent the weight of the [2 marks]
ball.

Markscheme
downward vertical arrow of any length
arrow of correct length

Judge the length of the vertical arrow by eye. The construction lines are not required. A
label is not required

eg:

[2 marks]

2c. Show that the magnitude of the net force F on the ball is given by the [3 marks]
following equation.

mg
F=
tan θ

64
Markscheme
ALTERNATIVE 1
F = N cos θ
mg = N sin θ
dividing/substituting to get result

ALTERNATIVE 2
right angle triangle drawn with F, N and W/mg labelled
angle correctly labelled and arrows on forces in correct directions
correct use of trigonometry leading to the required relationship

mg
tan θ = O
A
= F
[3 marks]

2d. The radius of the bowl is 8.0 m and θ = 22°. Determine the speed of the ball. [4 marks]

Markscheme
mg 2

tan θ
= m vr

r = R cos θ

v = √ sin θ /√ tan θ /√ 9.81×


gRcos2θ gR cos θ
8.0 cos 22
tan 22

v = 13.4/13 «ms –1»

Award [4] for a bald correct answer


Award [3] for an answer of 13.9/14 «ms –1». MP2 omitted
[4 marks]

2e. Outline whether this ball can move on a horizontal circular path of radius equal to [2 marks]
the radius of the bowl.

65
Markscheme
there is no force to balance the weight/N is horizontal
so no / it is not possible

Must see correct justification to award MP2


[2 marks]

2f. A second identical ball is placed at the bottom of the bowl and the first ball is [3 marks]
displaced so that its height from the horizontal is equal to 8.0 m.

The first ball is released and eventually strikes the second ball. The two balls remain in contact.
Determine, in m, the maximum height reached by the two balls.

Markscheme
speed before collision v = «√2gR =» 12.5 «ms–1»

«from conservation of momentum» common speed after collision is 12 initial speed «vc =
12.5
2
= 6.25 ms –1»
2 2
h = « 2gc = 26.25
v
×9.81
» 2.0 «m»

Allow 12.5 from incorrect use of kinematics equations


Award [3] for a bald correct answer
Award [0] for mg(8) = 2mgh leading to h = 4 m if done in one step.
Allow ECF from MP1
Allow ECF from MP2
[3 marks]

66 at a uniform speed.
A girl on a sledge is moving down a snow slope
A girl on a sledge is moving down a snow slope at a uniform speed.

3a. Draw the free-body diagram for the sledge at the position shown on the snow slope. [2 marks]

Markscheme
arrow vertically downwards labelled weight «of sledge and/or girl»/ W/mg/gravitational
force/Fg/Fgravitational AND arrow perpendicular to the snow slope labelled reaction
force/R/normal contact force/N/FN
friction force/F/f acting up slope «perpendicular to reaction force»
Do not allow G/g/“gravity”.
Do not award MP1 if a “driving force” is included.
Allow components of weight if correctly labelled.
Ignore point of application or shape of object.
Ignore “air resistance”.
Ignore any reference to “push of feet on sledge”.
Do not award MP2 for forces on sledge on horizontal ground
The arrows should contact the object

3b. After leaving the snow slope, the girl on the sledge moves over a horizontal region [3 marks]
of snow. Explain, with reference to the physical origin of the forces, why the vertical
forces on the girl must be in equilibrium as she moves over the horizontal region.

Markscheme
gravitational force/weight from the Earth «downwards»
reaction force from the sledge/snow/ground «upwards»
no vertical acceleration/remains in contact with the ground/does not move vertically as
there is no resultant vertical force
Allow naming of forces as in (a)
Allow vertical forces are balanced/equal in magnitude/cancel out

3c. When the sledge is moving on the horizontal region of the snow, the girl jumps off the [2 marks]
sledge. The girl has no horizontal velocity after the jump. The velocity of the sledge
immediately after the girl jumps off is 4.2 m s–1. The mass of the girl is 55 kg and the mass of
the sledge is 5.5 kg. Calculate the speed of the sledge immediately before the girl jumps from it.

67
Markscheme
mention of conservation of momentum
OR
5.5 x 4.2 = (55 + 5.5) «v»
0.38 «m s–1»
Allow p=p′ or other algebraically equivalent statement
Award [0] for answers based on energy

3d. The girl chooses to jump so that she lands on loosely-packed snow rather than frozen [3 marks]
ice. Outline why she chooses to land on the snow.

Markscheme
same change in momentum/impulse
the time taken «to stop» would be greater «with the snow»
Δp
F= Δt
therefore F is smaller «with the snow»

OR
force is proportional to rate of change of momentum therefore F is smaller «with the snow»
Allow reverse argument for ice

The sledge, without the girl on it, now travels up a snow slope that makes an angle of 6.5˚ to the
horizontal. At the start of the slope, the speed of the sledge is 4.2 m s–1. The coefficient of
dynamic friction of the sledge on the snow is 0.11.

3e. Show that the acceleration of the sledge is about –2 m s –2. [3 marks]

Markscheme
«friction force down slope» = µmg cos(6.5) = «5.9 N»
«component of weight down slope» = mg sin(6.5) «= 6.1 N»
12
F
«so a = m » acceleration = 5.5 = 2.2 «m s –2»

Ignore negative signs


Allow use of g = 10 m s –2

3f. Calculate the distance along the slope at which the sledge stops moving. Assume that [2 marks]
the coefficient of dynamic friction is constant.

68
Markscheme
correct use of kinematics equation
distance = 4.4 or 4.0 «m»
Alternative 2
KE lost=work done against friction + GPE
distance = 4.4 or 4.0 «m»
Allow ECF from (e)(i)
Allow [1 max] for GPE missing leading to 8.2 «m»

3g. The coefficient of static friction between the sledge and the snow is 0.14. Outline, [2 marks]
with a calculation, the subsequent motion of the sledge.

Markscheme
calculates a maximum value for the frictional force = « µR=» 7.5 «N»
sledge will not move as the maximum static friction force is greater than the component of
weight down the slope
Allow correct conclusion from incorrect MP1
Allow 7.5 > 6.1 so will not move

The diagram below shows part of a downhill ski course which starts at point A, 50 m above level
ground. Point B is 20 m above level ground.

A skier of mass 65 kg starts from rest at point A and during the ski course some of the
gravitational potential energy transferred to kinetic energy.

4a. From A to B, 24 % of the gravitational potential energy transferred to kinetic energy. [2 marks]
Show that the velocity at B is 12 m s–1.

69
Markscheme
1 2
2
v = 0.24 gh
v = 11.9 «m s –1»

Award GPE lost = 65 × 9.81 × 30 = «19130 J»


Must see the 11.9 value for MP2, not simply 12.
Allow g = 9.8 ms –2.

4b. Some of the gravitational potential energy transferred into internal energy of the skis, [2 marks]
slightly increasing their temperature. Distinguish between internal energy and
temperature.

Markscheme
internal energy is the total KE «and PE» of the molecules/particles/atoms in an object
temperature is a measure of the average KE of the molecules/particles/atoms

Award [1 max] if there is no mention of molecules/particles/atoms.

4c. The dot on the following diagram represents the skier as she passes point B. [2 marks]
Draw and label the vertical forces acting on the skier.

70
Markscheme
arrow vertically downwards from dot labelled weight/W/mg/gravitational
force/Fg/Fgravitational AND arrow vertically upwards from dot labelled reaction force/R/normal
contact force/N/FN
W>R

Do not allow gravity.


Do not award MP1 if additional ‘centripetal’ force arrow is added.
Arrows must connect to dot.
Ignore any horizontal arrow labelled friction.
Judge by eye for MP2. Arrows do not have to be correctly labelled or connect to dot for
MP2.

4d. The hill at point B has a circular shape with a radius of 20 m. Determine whether the [3 marks]
skier will lose contact with the ground at point B.

71
Markscheme
ALTERNATIVE 1
2
recognition that centripetal force is required / mv
r
seen

= 468 «N»
W/640 N (weight) is larger than the centripetal force required, so the skier does not lose
contact with the ground

ALTERNATIVE 2
v2
recognition that centripetal acceleration is required / r seen
a = 7.2 «ms –2»
g is larger than the centripetal acceleration required, so the skier does not lose contact with
the ground

ALTERNATIVE 3
recognition that to lose contact with the ground centripetal force ≥ weight
calculation that v ≥ 14 «ms –1»
comment that 12 «ms –1» is less than 14 «ms –1» so the skier does not lose contact with the
ground

ALTERNATIVE 4
mv2
recognition that centripetal force is required / r seen

calculation that reaction force = 172 «N»


reaction force > 0 so the skier does not lose contact with the ground

Do not award a mark for the bald statement that the skier does not lose contact with the
ground.

4e. The skier reaches point C with a speed of 8.2 m s –1. She stops after a distance of 24 [3 marks]
m at point D.
Determine the coefficient of dynamic friction between the base of the skis and the snow.
Assume that the frictional force is constant and that air resistance can be neglected.

72
Markscheme
ALTERNATIVE 1
0 = 8.2 2 + 2 × a × 24 therefore a = «−»1.40 «m s −2»
friction force = ma = 65 × 1.4 = 91 «N»
coefficient of friction = 65 919.81 = 0.14
×

ALTERNATIVE 2
KE = 12 mv2 = 0.5 x 65 x 8.2 2 = 2185 «J»

friction force = KE/distance = 2185/24 = 91 «N»


coefficient of friction = 65 919.81 = 0.14
×

Allow ECF from MP1.

At the side of the course flexible safety nets are used. Another skier of mass 76 kg falls normally
into the safety net with speed 9.6 m s–1.

4f. Calculate the impulse required from the net to stop the skier and state an appropriate [2 marks]
unit for your answer.

Markscheme
«76 × 9.6»= 730
Ns OR kg ms –1

4g. Explain, with reference to change in momentum, why a flexible safety net is less likely [2 marks]
to harm the skier than a rigid barrier.

Markscheme
safety net extends stopping time
Δp
F = Δt therefore F is smaller «with safety net»

OR
force is proportional to rate of change of momentum therefore F is smaller «with safety net»

Accept reverse argument.

73
A glider is an aircraft with no engine. To be launched, a glider is uniformly accelerated from rest
A glider is an aircraft with no engine. To be launched, a glider is uniformly accelerated from rest
by a cable pulled by a motor that exerts a horizontal force on the glider throughout the launch.

5a. The glider reaches its launch speed of 27.0 m s –1 after accelerating for 11.0 s. Assume[2 marks]
that the glider moves horizontally until it leaves the ground. Calculate the total distance
travelled by the glider before it leaves the ground.

Markscheme
correct use of kinematic equation/equations
148.5 or 149 or 150 «m»

Substitution(s) must be correct.

5b. The glider and pilot have a total mass of 492 kg. During the acceleration the glider is [3 marks]
subject to an average resistive force of 160 N. Determine the average tension in the
cable as the glider accelerates.

Markscheme
a = 27
11
or 2.45 «m s –2»

F – 160 = 492 × 2.45


1370 «N»

Could be seen in part (a).


Award [0] for solution that uses a = 9.81 m s –2

5c. The cable is pulled by an electric motor. The motor has an overall efficiency of 23 %. [3 marks]
Determine the average power input to the motor.

74
Markscheme
ALTERNATIVE 1
«work done to launch glider» = 1370 x 149 «= 204 kJ»
«work done by motor» = 20423
×100

«power input to motor» = 20423


×100 × 1 = 80 or 80.4 or 81 k«W»
11

ALTERNATIVE 2
use of average speed 13.5 m s –1
«useful power output» = force x average speed «= 1370 x 13.5»
power input = «1370 × 13.5 × 100
23
=» 80 or 80.4 or 81 k«W»

ALTERNATIVE 3
work required from motor = KE + work done against friction «
= 0.5 × 492 × 272 + (160 × 148.5)» = 204 «kJ»
work required from motor×100
«energy input» = 23

power input = 883000


11
= 80.3 k«W»

Award [2 max] for an answer of 160 k«W».

5d. The cable is wound onto a cylinder of diameter 1.2 m. Calculate the angular velocity of [2 marks]
the cylinder at the instant when the glider has a speed of 27 m s–1. Include
an appropriate unit for your answer.

Markscheme
27
ω = « vr =» 0.6
= 45
rad s –1

Do not accept Hz.


Award [1 max] if unit is missing.

75
After takeoff the cable is released and the unpowered glider moves horizontally
5e. After takeoff the cable is released and the unpowered glider moves horizontally [2 marks]
at constant speed. The wings of the glider provide a lift force. The diagram shows
the lift force acting on the glider and the direction of motion of the glider.

Draw the forces acting on the glider to complete the free-body diagram. The dotted lines show
the horizontal and vertical directions.

Markscheme

drag correctly labelled and in correct direction


weight correctly labelled and in correct direction AND no other incorrect force shown

Award [1 max] if forces do not touch the dot, but are otherwise OK.

5f. Explain, using appropriate laws of motion, how the forces acting on the glider [2 marks]
maintain it in level flight.

Markscheme
name Newton's first law
vertical/all forces are in equilibrium/balanced/add to zero
OR
vertical component of lift mentioned
as equal to weight

At a particular instant in the flight the glider is76


losing 1.00 m of vertical height for every
5g. At a particular instant in the flight the glider is losing 1.00 m of vertical height for every [3 marks]
6.00 m that it goes forward horizontally. At this instant, the horizontal speed of the
glider is 12.5 m s–1. Calculate the velocity of the glider. Give your answer to an appropriate
number of significant figures.

Markscheme
any speed and any direction quoted together as the answer
quotes their answer(s) to 3 significant figures
speed = 12.7 m s –1 or direction = 9.46 º or 0.165 rad «below the horizontal» or gradient of
− 16

A company designs a spring system for loading ice blocks onto a truck. The ice block is placed
in a holder H in front of the spring and an electric motor compresses the spring by pushing H to
the left. When the spring is released the ice block is accelerated towards a
ramp ABC. When the spring is fully decompressed, the ice block loses contact with the spring at
A. The mass of the ice block is 55 kg.

Assume that the surface of the ramp is frictionless and that the masses of the spring and the
holder are negligible compared to the mass of the ice block.

6a. (i) The block arrives at C with a speed of 0.90ms −1. Show that the elastic energy [4 marks]
stored in the spring is 670J.
(ii) Calculate the speed of the block at A.

77
Markscheme
(i)
≪ Eel= ≫ 12 mv2 + mgh
OR
«Eel=»EP+EK
1
≪ Eel= ≫ 2
× 55 × 0.902 +55 × 9.8 × 1.2
OR
669 J
«Eel = 669 ≈ 670J»
Award [1 max] for use of g=10Nkg –1, gives 682 J.
(ii)
1
2
× 55 × v2 = 670J

v =≪ √ 2×55670 = ≫ 4.9ms−1

If 682J used, answer is 5.0ms –1.

6b. Describe the motion of the block [3 marks]


(i) from A to B with reference to Newton's first law.
(ii) from B to C with reference to Newton's second law.

Markscheme
(i)
no force/friction on the block, hence constant motion/velocity/speed
(ii)
force acts on block OR gravity/component of weight pulls down slope
velocity/speed decreases OR it is slowing down OR it decelerates
Do not allow a bald statement of “N2” or “F = ma” for MP1.
Treat references to energy as neutral.

78
On the axes, sketch a graph to show how the displacement of the block varies with
6c. On the axes, sketch a graph to show how the displacement of the block varies with [2 marks]
time from A to C. (You do not have to put numbers on the axes.)

Markscheme
straight line through origin for at least one-third of the total length of time axis covered by
candidate line
followed by curve with decreasing positive gradient

Ignore any attempt to include motion before A.


Gradient of curve must always be less than that of straight line.

6d. The spring decompression takes 0.42s. Determine the average force that the spring [2 marks]
exerts on the block.

Markscheme
Δp 55×4.9
F ≪= Δt
≫= 0.42
F=642≈640N
Allow ECF from (a)(ii).

79potential difference 120V and draws a


The electric motor is connected to a source of
6e. The electric motor is connected to a source of potential difference 120V and draws a [2 marks]
current of 6.8A. The motor takes 1.5s to compress the spring.
Estimate the efficiency of the motor.

Markscheme
«energy supplied by motor =» 120 × 6.8 × 1.5 or 1224 J
OR
«power supplied by motor =» 120 × 6.8 or 816 W
e = 0.55 or 0.547 or 55% or 54.7%
Allow ECF from earlier results.

© International Baccalaureate Organization 2019


International Baccalaureate® - Baccalauréat International® - Bachillerato Internacional®

Printed for Concordian International School

80

You might also like